Sanggahan untuk “200 Bukti Bumi Datar” dari Eric Dubay

Eric Dubay membuat sebuah daftar yang dia klaim sebagai “200 bukti Bumi datar.” Berikut adalah daftar dari “bukti-bukti” tersebut beserta sanggahannya dalam bentuk penjelasan singkat atau taut ke penjelasan kami di situs ini.

“Bukti-bukti” Dubay adalah hasil terjemahan Google Translate sehingga terjemahannya tidak sempurna.

Daftar ini masih dalam pengembangan. Sanggahan-sanggahan baru akan selalu ditambahkan secara berkala.

Horizon (1-2)

1Cakrawala selalu tampak sangat datar 360 derajat di sekitar pengamat tanpa memandang ketinggian. Semua balon amatir, roket, pesawat dan cuplikan drone menunjukkan horison yang benar-benar datar dengan tinggi lebih dari 20+ mil. Hanya NASA dan “lembaga antariksa” pemerintah lainnya yang menunjukkan kelengkungan dalam foto / video CGI palsu mereka.

2Cakrawala selalu naik ke tingkat mata pengamat saat ketinggian diperoleh, jadi Anda tidak perlu melihat ke bawah untuk melihatnya.Jika Bumi ternyata adalah bola bumi, tidak peduli seberapa besar, ketika Anda naik cakrawala akan tetap diperbaiki dan pengamat / kamera harus memiringkan melihat ke bawah lebih jauh dan lebih jauh untuk melihatnya.

Air, Sungai & Terusan (3-9)

3Fisika alami air adalah untuk menemukan dan mempertahankan levelnya. Jika Bumi adalah bola raksasa yang miring, terhuyung-huyung dan terguncang melalui ruang tanpa batas, maka permukaan datar benar-benar datar tidak akan ada di sini. Tapi karena Bumi sebenarnya adalah sebuah bidang datar yang panjang, sifat fisik fundamental dari cairan yang ditemukan dan tingkat yang tersisa ini konsisten dengan pengalaman dan akal sehat.”

4Sungai-sungai mengalir ke permukaan laut menemukan jalan termudah, Utara, Selatan, Timur, Barat dan semua arah perantara lainnya di atas Bumi pada saat yang bersamaan. Jika Bumi benar-benar bola yang berputar maka banyak dari sungai-sungai ini akan mengalir tanpa hambatan ke atas, misalnya Mississippi dalam 3000 milnya harus naik 11 mil sebelum mencapai Teluk Meksiko.

5Satu bagian dari Sungai Nil mengalir sejauh seribu mil dengan jatuhnya hanya satu kaki. Bagian dari Kongo Afrika Barat, sesuai dengan kecenderungan dan gerakan bola Bumi, kadang-kadang akan berjalan menanjak dan kadang-kadang turun. Ini juga akan menjadi kasus untuk Parana, Paraguay, dan sungai panjang lainnya.

6Jika Bumi adalah bola 25.000 mil dalam keliling sebagai NASA dan klaim astronomi modern, trigonometri bola menentukan permukaan semua air yang berdiri harus melengkung ke bawah yang mudah diukur 8 inci per mil dikalikan dengan kuadrat jarak. Ini berarti sepanjang 6 mil saluran air berdiri, Bumi akan mencelup 6 kaki di kedua ujung dari puncak pusat. Setiap kali percobaan seperti itu telah dilakukan, bagaimanapun, air berdiri telah terbukti sempurna.”

7Surveyor, insinyur dan arsitek tidak pernah diperlukan untuk faktor kelengkungan yang seharusnya Bumi ke dalam proyek-proyek mereka. Kanal, rel kereta api, jembatan dan terowongan misalnya selalu dipotong dan diletakkan secara horizontal, seringkali lebih dari ratusan mil tanpa ada kelonggaran.

8Terusan Suez yang menghubungkan Laut Tengah dengan Laut Merah adalah 100 mil panjang tanpa kunci yang membuat air menjadi kelanjutan tak terputus dari dua lautan. Ketika dibangun, kelengkungan bumi seharusnya tidak diperhitungkan, itu digali sepanjang garis datum horisontal 26 kaki di bawah permukaan laut, melewati beberapa danau dari satu laut ke laut lainnya, dengan garis datum dan permukaan air berjalan paralel sempurna atas 100 mil. 

9Insinyur, W. Winckler dipublikasikan di Earth Review mengenai kelengkungan bumi, menyatakan, “Sebagai seorang insinyur selama bertahun-tahun berdiri, saya melihat bahwa tunjangan yang absurd ini hanya diizinkan di buku sekolah. Insinyur tidak akan bermimpi mengizinkan hal semacam itu. Saya telah memproyeksikan bermil-mil jalan kereta api dan banyak lagi kanal dan uang saku bahkan belum terpikirkan, apalagi diizinkan. Tunjangan kelengkungan ini berarti ini – bahwa itu adalah 8 “untuk mil pertama dari sebuah kanal, dan meningkat pada rasio dengan kuadrat jarak dalam mil;jadi saluran kecil yang bisa dilayari untuk kapal, katakanlah sepanjang 30 mil, akan memiliki, dengan aturan di atas, penyisihan kelengkungan 600 kaki. Pikirkan itu dan kemudian harap para insinyur kredit tidak terlalu bodoh. Hal semacam itu tidak diizinkan.Kami tidak lagi berpikir untuk memungkinkan 600 kaki untuk garis 30 mil dari jalur kereta api atau kanal, daripada membuang-buang waktu kami mencoba membuat lingkaran persegi ” “

Rel Kereta (10-12)

10London dan Northwestern Railway membentuk garis lurus 180 mil panjang antara London dan Liverpool. Titik tertinggi kereta api, di tengah-tengah di stasiun Birmingham, hanya 240 meter di atas permukaan laut. Jika dunia sebenarnya adalah dunia, bagaimanapun, melengkung 8 inci per mil kuadrat, bentangan 180 mil rel akan membentuk busur dengan titik pusat di Birmingham meningkatkan lebih dari satu mil, yang penuh 5.400 meter di atas London dan Liverpool.”

11Seorang surveyor dan insinyur dari tiga puluh tahun yang diterbitkan di Birmingham Weekly Mercury menyatakan, “Saya benar-benar memahami teori dan praktek teknik sipil. Namun sebagian besar dari para profesor kami mungkin berada dalam teori survei sesuai dengan aturan yang ditentukan, namun sangat terkenal di antara kita bahwa pengukuran teoritis seperti itu MUNGKIN DARI ILUSTRASI PRAKTIS. Semua lokomotif kami dirancang untuk berjalan pada apa yang dapat dianggap sebagai TINGKAT TRUE atau FLATS.Tentu saja ada kemiringan atau kemiringan parsial di sana-sini, tetapi mereka selalu didefinisikan secara akurat dan harus dilacak secara hati-hati. Tapi apapun yang mendekati delapan inci di mil, meningkat sebagai kuadrat jarak, TIDAK BISA DILAKUKAN OLEH MESIN YANG PERNAH DITETAPKAN. Mengambil satu stasiun dengan yang lain di seluruh Inggris dan Skotlandia, dapat dinyatakan bahwa semua platform BERADA PADA TINGKAT RELATIF YANG SAMA. Jarak antara Timur dan Barat pantai Inggris dapat ditetapkan sebagai 300 mil.Jika kelengkungan yang ditentukan memang terwakili, stasiun pusat di Rugby atau Warwick seharusnya dekat tiga mil lebih tinggi dari akor yang ditarik dari dua ekstremitas. Jika demikian halnya, tidak ada pengemudi atau penjarah dalam Kerajaan yang akan ditemukan untuk mengambil alih kereta. Kami hanya bisa menertawakan para pembaca Anda yang dengan serius memberi kami penghargaan atas berbagai eksploitasi yang berani, karena berlari melintasi kurva bulat. Kurva horizontal pada level cukup berbahaya, kurva vertikal akan menjadi seribu kali lebih buruk, dan dengan rolling stock kami dibangun seperti saat ini secara fisik tidak mungkin. ”

12Perusahaan Kanal Kapal Manchester yang diterbitkan dalam Earth Review menyatakan, “Ini adalah kebiasaan dalam konstruksi Kereta Api dan Kanal untuk semua tingkatan untuk dirujuk ke sebuah datum yang secara horisontal nominal dan ditampilkan di semua bagian.Bukanlah praktek dalam membuat Pekerjaan Umum untuk membuat kelonggaran atas kelengkungan bumi”

Keterlihatan Objek Jauh (13-14)

13Dalam percobaan Perancis abad ke-19 oleh MM Biot dan Arago, sebuah lampu yang kuat dengan reflektor yang baik ditempatkan di puncak Desierto las Palmas di Spanyol dan dapat dilihat sepanjang jalan dari Camprey di Pulau Iviza. Karena ketinggian kedua titik itu identik dan jarak antara hampir 100 mil, jika Bumi adalah lingkaran 25.000 mil, maka seharusnya lebih dari 6.600 kaki, satu mil dan seperempat, di bawah garis pandang!”

14Eksperimen Letnan-Kolonel Portlock menggunakan lampu oxy-hydrogen Drummond dan heliostats untuk memantulkan sinar matahari di seluruh stasiun yang dipasang di 108 mil dari St. George’s Channel. Jika Bumi benar-benar bola 25.000 mil di lingkar, cahaya Portlock seharusnya tetap tersembunyi di bawah satu mil dan setengah kelengkungan.”

Lain-Lain (15-19)

15Jika Bumi benar-benar lingkaran lingkaran 25.000 mil, pilot pesawat harus terus-menerus mengoreksi ketinggian mereka ke bawah sehingga tidak langsung terbang ke “angkasa luar;” seorang pilot yang ingin mempertahankan ketinggian mereka pada kecepatan jelajah yang khas. dari 500 mph, harus terus mencelupkan hidung mereka ke bawah dan turun 2,777 kaki (lebih dari setengah mil) setiap menit! Kalau tidak, tanpa kompensasi, dalam waktu satu jam, pilot akan menemukan diri mereka 31,5 mil lebih tinggi dari yang diperkirakan.

16Percobaan yang dikenal sebagai “Kegagalan Airy” membuktikan bahwa bintang bergerak relatif terhadap Bumi stasioner dan bukan sebaliknya. Dengan terlebih dahulu mengisi teleskop dengan air untuk memperlambat kecepatan cahaya di dalam, kemudian menghitung kemiringan yang diperlukan untuk mendapatkan cahaya bintang langsung ke bawah tabung, Airy gagal membuktikan teori heliosentris sejak cahaya bintang sudah datang di sudut yang benar tanpa perubahan diperlukan, dan bukannya membuktikan model geosentris yang benar.

17“Olber’s Paradox” menyatakan bahwa jika ada miliaran bintang yang sedang matahari langit malam akan diisi sepenuhnya dengan cahaya. Seperti Edgar Allen Poe berkata, “Apakah suksesi bintang-bintang tak berujung, maka latar belakang langit akan menghadirkan kita luminositas yang seragam, karena sama sekali tidak ada titik, di semua latar belakang itu, di mana tidak akan ada bintang.” Fakta “Paradox” Olber tidak lebih merupakan paradoks dari eksperimen George Airy adalah “kegagalan”. Keduanya sebenarnya adalah sanggahan yang sangat baik dari model bola berputar heliosentris.

18Percobaan Michelson-Morley dan Sagnac berusaha mengukur perubahan dalam kecepatan cahaya karena gerak yang diasumsikan Bumi melalui ruang angkasa. Setelah mengukur dalam setiap kemungkinan arah yang berbeda di berbagai lokasi, mereka gagal mendeteksi perubahan signifikan apa pun, sekali lagi membuktikan model geosentris stasioner.

19Tycho Brahe terkenal menentang teori heliosentris pada masanya, dengan menyatakan bahwa jika Bumi berputar mengelilingi Matahari, perubahan posisi relatif bintang-bintang setelah gerakan orbital 6 bulan tidak dapat gagal untuk dilihat. Dia berpendapat bahwa bintang-bintang harus tampak terpisah ketika kita mendekati dan datang bersama saat kita surut. Namun kenyataannya, bagaimanapun, setelah 190.000.000 mil yang diperkirakan mengorbit Matahari, tidak satu pun parallax dapat dideteksi di bintang-bintang, membuktikan kita tidak bergerak sama sekali.

Rotasi Bumi (20-31)

20Jika Bumi benar-benar terus berputar ke arah timur di atas 1000mph, cannonballs yang dipicu secara vertikal dan proyektil lainnya harus jatuh secara signifikan ke arah barat. Namun, pada kenyataannya, setiap kali ini telah diuji, cannonballs yang ditembakkan secara vertikal menembak ke atas rata-rata 14 detik naik, 14 detik turun, dan jatuh kembali ke tanah tidak lebih dari 2 kaki dari meriam, sering langsung kembali ke moncong.

21 “Jika Bumi benar-benar terus berputar ke arah timur dengan kecepatan lebih dari 1000mph, helikopter dan balon udara harus dapat melayang-layang di atas permukaan Bumi dan menunggu sampai tujuan mereka datang kepada mereka”

22 “Jika Bumi benar-benar terus berputar ke arah timur di lebih dari 1000mph, selama menyelam Red Bull stratosfer, Felix Baumgartner, menghabiskan 3 jam naik di atas New Mexico, seharusnya telah mendaratkan 2.500 mil Barat ke Samudra Pasifik tetapi malah mendaratkan beberapa lusin mil ke Timur dari titik take-off.”

23Para penganut bola kerap mengklaim “gravitasi” secara ajaib dan tanpa dapat dijelaskan menyeret seluruh atmosfer bumi yang lebih rendah dalam sinkronisasi sempurna hingga ketinggian yang belum ditentukan di mana atmosfer berputar yang semakin cepat ini memberi jalan kepada non-pemintalan, non-gravitasi, non- atmosfer ruang hampa yang tak terbatas. Teori non-sensis seperti itu dibantah, bagaimanapun, oleh hujan, kembang api, burung, serangga, awan, asap, pesawat dan proyektil yang semuanya akan berperilaku sangat berbeda jika kedua bola-Bumi dan atmosfernya terus berputar ke arah timur pada 1000mph.

24Jika Bumi dan atmosfernya secara konstan berputar ke arah timur lebih dari 1000mph maka meriam Utara / Selatan menghadap harus membuat kontrol sementara tembakan meriam menembak-Timur harus jatuh lebih jauh daripada yang lain, sementara cannonballs yang menembakkan ke arah barat harus jatuh lebih dekat. Namun, dalam kenyataannya, terlepas dari arah mana meriam yang ditembakkan, jarak yang ditempuh selalu sama.

25Jika Bumi dan atmosfernya terus berputar ke arah timur lebih dari 1000mph, maka rata-rata pesawat komersial yang bepergian sejauh 500mph tidak akan pernah bisa mencapai tujuan ke arah Timur sebelum mereka datang dari belakang! Demikian juga tujuan ke arah barat harus tiba dengan kecepatan tiga kali lipat, tetapi bukan ini masalahnya.

26Mengutip” Surga dan Bumi “oleh Gabrielle Henriet,” Jika terbang telah ditemukan pada saat Copernicus, tidak ada keraguan bahwa dia akan segera menyadari bahwa pendapatnya mengenai rotasi bumi itu salah, karena hubungan yang ada antara kecepatan pesawat terbang dan rotasi bumi. Jika bumi berputar, seperti yang dikatakan, pada 1.000 mil per jam, dan sebuah pesawat terbang ke arah yang sama pada hanya 500 mil, jelas bahwa tempat tujuannya akan lebih jauh dihapus setiap menit. Di sisi lain, jika terbang terjadi pada arah berlawanan dengan rotasi, jarak 1.500 mil akan tertutup dalam satu jam, bukannya 500, karena kecepatan rotasi akan ditambahkan ke pesawat . Bisa juga ditunjukkan bahwa kecepatan terbang sebesar 1.000 mil per jam, yang seharusnya menjadi rotasi bumi, baru-baru ini tercapai, sehingga pesawat terbang terbang pada tingkat ini dengan arah yang sama dengan rotasi tidak bisa menutupi tanah sama sekali. Itu akan tetap tergantung di udara di atas tempat dari mana ia lepas landas, karena kedua kecepatan sama.

27 “Jika Bumi dan atmosfernya secara konstan berputar ke arah Timur lebih dari 1000mph, mendaratkan pesawat di landasan pacu yang bergerak cepat yang menghadapi segala macam arah Utara, Selatan, Timur, Barat dan sebaliknya akan hampir tidak mungkin, namun pada kenyataannya kekhawatiran fiktif tersebut benar-benar dapat diabaikan.”

28 “Jika Bumi dan atmosfernya secara konstan berputar ke arah Timur lebih dari 1000mph, maka awan, angin dan pola cuaca tidak bisa dengan santai dan tak terduga pergi ke mana pun, dengan awan sering bepergian ke arah berlawanan di berbagai ketinggian secara bersamaan.”

29Jika Bumi dan atmosfernya secara konstan berputar ke arah Timur lebih dari 1000mph, ini entah bagaimana harus dilihat, didengar, dirasakan, atau diukur oleh seseorang, namun tidak seorang pun dalam sejarah pernah mengalami gerakan ke arah Timur ini; sementara itu, kita dapat mendengar, merasakan, dan secara eksperimental mengukur bahkan sedikit angin Westward.”

30Dalam bukunya” South Sea Voyages, “Penjelajah Kutub Utara dan Antartika Sir James Clarke Ross, menggambarkan pengalamannya pada malam 27 November 1839 dan kesimpulannya bahwa Bumi harus tidak bergerak:” Langit menjadi sangat jelas … itu memungkinkan kita untuk mengamati lapisan awan yang lebih tinggi untuk bergerak dalam arah yang berlawanan dengan angin — suatu keadaan yang sering dicatat dalam jurnal meteorologi kita baik di perdagangan timur laut dan tenggara, dan juga sering diamati. oleh mantan pelayar. Kapten Basil Hall menyaksikannya dari puncak Puncak Teneriffe; dan Hitung Strzelechi, saat mendaki gunung vulkanik Kiranea, di Owhyhee, mencapai ketinggian 4000 kaki di atas angin perdagangan, dan mengalami pengaruh arus udara berlawanan dari kondisi hygrometrik dan termometrik yang berbeda … Count Strzelechi lebih lanjut menginformasikan saya dari keadaan yang tampaknya anomali berikut ini — bahwa pada ketinggian 6000 kaki dia menemukan arus hembusan udara pada sudut siku-siku ke lapisan bawah, juga kondisi higrometrik dan termometri yang berbeda, tetapi lebih hangat daripada antar-lapisan. Keadaan atmosfer seperti itu hanya kompatibel dengan fakta yang telah ditunjukkan oleh bukti lain, bahwa bumi sedang beristirahat. ”

  • TBD(arah-awan)

31Mengutip” Zetetic Cosmogeny “Thomas Winships menyatakan:” Biarkan ‘imajinasi’ membayangkan pikiran apa kekuatan udara yang digerakkan oleh tubuh bulat berdiameter 8.000 mil, yang dalam satu jam berputar sepanjang 1.000 mph, bergegas melintasi ruang dengan kecepatan 65.000 mph dan berputar di langit? Kalau begitu, mari kita ‘berspekulasi’ usaha untuk menemukan apakah penduduk di dunia seperti itu bisa menjaga rambut mereka? Jika bumi-bumi berputar pada porosnya pada kecepatan hebat 1.000 mil per jam, massa yang sangat besar akan menyebabkan hembusan angin yang luar biasa di ruang yang ditempati. Angin akan pergi ke satu arah, dan apa pun seperti awan yang masuk ‘dalam lingkup pengaruh’ dari bola yang berputar, harus pergi dengan cara yang sama.Kenyataan bahwa bumi sedang beristirahat dibuktikan dengan terbang layang-layang.”

Gravitasi (32-33)

32 “Jika“ gravitasi ”dikreditkan dengan kekuatan yang cukup kuat untuk menahan lautan, bangunan, orang, dan atmosfer dunia yang menempel di permukaan bola yang berputar cepat, maka mustahil bagi“ gravitasi ”juga secara bersamaan menjadi cukup lemah untuk memungkinkan burung kecil, serangga, dan pesawat untuk lepas landas dan melakukan perjalanan dengan bebas di semua arah. ”

33 “”Jika” gravitasi “dikreditkan dengan kekuatan yang cukup kuat untuk melengkungkan hamparan luas lautan di sekitar Bumi yang bulat, tidak mungkin bagi ikan dan makhluk lain untuk berenang melalui air yang dipegang kuat seperti itu.”

Navigasi Kapal (34-42)

34 “Kapten kapal dalam menavigasi jarak yang jauh di laut tidak pernah perlu faktor kelengkungan Bumi seharusnya menjadi perhitungan mereka.Baik Plane Sailing dan Great Circle Sailing, metode navigasi yang paling populer, menggunakan pesawat, bukan trigonometri bulat, membuat semua perhitungan matematis dengan asumsi bahwa Bumi sangat datar.Jika Bumi ternyata adalah bola, asumsi yang keliru seperti itu akan menyebabkan ketidakakuratan yang mencolok.Plane Sailing telah bekerja dengan baik baik dalam teori maupun praktek selama ribuan tahun, dan pesawat trigonometri telah terbukti berkali-kali lebih akurat daripada trigonometri bola dalam menentukan jarak di lautan. ”

35Jika Bumi benar-benar bola dunia, maka setiap garis lintang selatan dari khatulistiwa harus mengukur keliling yang secara bertahap lebih kecil dan lebih kecil semakin jauh yang dilalui Selatan. Namun, jika Bumi adalah bidang yang diperluas, maka setiap garis lintang selatan dari khatulistiwa harus mengukur keliling yang secara bertahap lebih besar dan lebih besar, semakin jauh yang dilalui Selatan. Fakta bahwa banyak kapten yang menavigasi ke selatan khatulistiwa dengan asumsi teori globular telah menemukan diri mereka secara drastis tidak diperhitungkan, lebih jauh lagi perjalanan Selatan, membuktikan fakta bahwa Bumi bukanlah bola. ”

36 “Selama perjalanan Kapten James Clark Ross di sekitar lingkar Antartika, dia sering menulis dalam jurnalnya bingung bagaimana mereka secara rutin menemukan diri mereka sesuai dengan grafik mereka, menyatakan bahwa mereka menemukan diri mereka rata-rata 12-16 mil di luar perhitungan mereka setiap hari , selanjutnya di selatan sejauh 29 mil. “

  • Spekulasi. Hal tersebut tidak ada kaitannya dengan bentuk Bumi, dan tidak perlu bantahan.

37 “Letnan Charles Wilkes memerintahkan ekspedisi eksplorasi Angkatan Laut Amerika Serikat ke Antartika dari tahun 1838 hingga 1842, dan dalam jurnal-jurnalnya juga disebutkan secara konsisten timur dari hisabnya, kadang-kadang lebih dari 20 mil dalam waktu kurang dari 18 jam.”

  • Spekulasi. Hal tersebut tidak ada kaitannya dengan bentuk Bumi, dan tidak perlu bantahan.

38 “Mengutip Pendeta Thomas Milner,“ Di belahan bumi selatan, para navigator ke India sering membayangkan diri mereka di timur Cape ketika masih di barat, dan telah terdampar di pantai Afrika, yang menurut perhitungan mereka, berada di belakang mereka.Kemalangan ini terjadi pada frigate yang bagus, Challenger, pada tahun 1845. Bagaimana datangnya Kapal Perdamaian Yang Mulia, ‘hilang?Bagaimana begitu banyak kapal bangsawan lainnya, suara sempurna, sempurna diawaki dengan sempurna, telah dirusak dalam cuaca yang tenang, tidak hanya di malam yang gelap, atau dalam kabut, tetapi di siang bolong dan sinar matahari – dalam kasus pertama di pantai, di yang terakhir, di atas bebatuan cekung – dari ‘keluar dari perhitungan?’ ”Jawaban sederhananya adalah Bumi bukanlah bola.”

  • Spekulasi. Hal tersebut tidak ada kaitannya dengan bentuk Bumi, dan tidak perlu bantahan.

39Pengukuran jarak praktis yang diambil dari“ The Australian Handbook, Almanack, Shippers ‘dan Importir’ Directory ”menyatakan bahwa jarak garis lurus antara Sydney dan Nelson adalah 1550 mil undang-undang. Perbedaan yang diberikan mereka dalam garis bujur adalah 22 derajat 2’14 “.Oleh karena itu jika 22 derajat 2’14 “dari 360 adalah 1550 mil, keseluruhan akan mengukur 25.182 mil. Ini tidak hanya lebih besar dari bola-Bumi dikatakan berada di khatulistiwa, tetapi keseluruhan 4262 mil lebih besar daripada di lintang selatan Sydney di dunia dengan proporsi seperti itu. “

  • Nelson dan Sydney terletak di garis lintang yang berbeda.
  • Jarak yang dikutip seharusnya merupakan jarak great-circle, bukan jarak longitudinal.
  • Jarak yang dikutip salah, seharusnya 1307 mil, bukan 1550 mil.

40 “Dari dekat Cape Horn, Chili ke Port Philip di Melbourne, Australia jaraknya adalah 10.500 mil, atau 143 derajat bujur jauhnya.Anjak dalam derajat yang tersisa hingga 360 membuat total jarak 26,430 mil di sekitar garis lintang khusus ini, yang lebih dari 1500 mil lebih lebar dari Bumi seharusnya berada di khatulistiwa, dan banyak lagi ribuan mil lebih lebar daripada seharusnya berada di lintang Selatan seperti itu. “

  • Jarak great-circle dari Cape Horn ke Melbourne seharusnya adalah 5.781 mil, bukan 10500 mil.

41 “Perhitungan serupa yang dibuat dari Tanjung Harapan, Afrika Selatan ke Melbourne, Australia pada garis lintang rata-rata 35,5 derajat Selatan, telah memberikan angka perkiraan lebih dari 25.000 mil, yang lagi sama dengan atau lebih besar dari lingkar bumi terbesar yang pernah ada di Bumi. di khatulistiwa.Perhitungan dari Sydney, Australia ke Wellington, Selandia Baru pada rata-rata 37,5 derajat Selatan telah memberikan keliling perkiraan 25.500 mil, lebih besar lagi!Menurut teori Ball-Earth, keliling Bumi pada 37,5 derajat Lintang Selatan seharusnya hanya 19.757 kilometer undang-undang, hampir enam ribu mil lebih kecil daripada ukuran praktis seperti itu. ”

  • Dubay tidak memberikan perhitungannya, dan tidak mungkin untuk mengetahui dimana kesalahannya.

42Dalam bola-Bumi model Antartika adalah benua es yang menutupi bagian bawah bola dari 78 derajat Lintang Selatan hingga 90 dan karena itu tidak lebih dari 12.000 mil di lingkar. Banyak penjelajah awal termasuk Captian Cook dan James Clark Ross, bagaimanapun, dalam upaya perjalanan mengelilingi Antartika memakan waktu 3 hingga 4 tahun dan mencatat waktu sekitar 50-60.000 mil. Kapal Inggris Challenger juga melakukan perjalanan tidak langsung tetapi mengelilingi Antartika melintasi 69.000 mil. Ini sepenuhnya tidak konsisten dengan model bola. ”

Rute Penerbangan Selatan (43-48)

43 “Jika Bumi adalah bola, ada beberapa penerbangan di belahan bumi Selatan yang akan memiliki jalur tercepat dan paling lurus di benua Antartika seperti Santiago, Chili ke Sydney, Australia.Alih-alih mengambil rute terpendek dan tercepat dalam garis lurus di atas Antartika, semua penerbangan seperti itu menjauhkan segala arah dari Antartika bukannya mengklaim suhu terlalu dingin untuk perjalanan pesawat!Mempertimbangkan fakta bahwa ada banyak penerbangan ke / dari / di atas Antartika, dan NASA mengklaim memiliki teknologi yang menjaga mereka dalam kondisi yang jauh lebih dingin (dan jauh lebih panas) daripada yang pernah dialami di Bumi, alasan semacam itu jelas hanya sebuah alasan, dan ini penerbangan tidak dibuat karena tidak mungkin. ”

44Jika Bumi adalah sebuah bola, dan Antartika terlalu dingin untuk terbang, satu-satunya cara yang logis untuk terbang dari Sydney ke Santiago akan menjadi tembakan lurus melintasi Pasifik yang tinggal di belahan bumi Selatan sepanjang jalan. Pengisian ulang dapat dilakukan di Selandia Baru atau tujuan belahan bumi Selatan lainnya sepanjang jalan jika benar-benar diperlukan. Kenyataannya, bagaimanapun, penerbangan Santiago-Sydney pergi ke belahan bumi Utara membuat penghentian di LAX dan bandara Amerika Utara lainnya sebelum melanjutkan kembali ke belahan bumi Selatan.Jalan memutar konyol seperti itu tidak masuk akal di dunia tetapi masuk akal dan membentuk garis hampir lurus ketika ditampilkan pada peta Bumi yang datar. ”

45 “Pada bola-Bumi, Johannesburg, Afrika Selatan ke Perth, Australia harus menjadi tembakan langsung di atas Samudera Hindia dengan kemungkinan pengisian ulang yang mudah di Mauritus atau Madagaskar.Namun, dalam praktik sebenarnya, sebagian besar penerbangan Johannesburg ke Perth dengan penuh rasa ingin tahu berhenti di Dubai, Hong Kong, atau Malaysia, semuanya tidak masuk akal pada bola, tetapi benar-benar dapat dimengerti ketika memetakan Bumi yang datar. ”

46 “Pada bola-Earth Cape Town, Afrika Selatan ke Buenos Aries, Argentina harus lurus ke Atlantik mengikuti garis lintang yang sama, tetapi setiap penerbangan pergi ke lokasi penghubung di belahan bumi utara terlebih dahulu, berhenti di mana saja dari London ke Turki ke Dubai.Sekali lagi ini benar-benar tidak masuk akal di dunia tetapi merupakan pilihan yang sepenuhnya bisa dimengerti ketika dipetakan di Bumi yang datar. ”

47 “Pada bola-Bumi Johannesburg, Afrika Selatan ke Sao Paolo, Brasil harus menjadi tembakan lurus cepat di sepanjang 25 lintang Selatan, tetapi sebagai gantinya hampir setiap penerbangan membuat pemberhentian bahan bakar kembali di tingkat ke-50 Lintang Utara di London lebih dulu!Satu-satunya alasan berhenti bekerja yang konyol dalam kenyataannya adalah karena Bumi itu datar. ”

48 “Pada bola-Bumi Santiago, Chili ke Johannesburg, Afrika Selatan harus menjadi penerbangan mudah semua berlangsung di bawah Tropic of Capricorn di belahan bumi Selatan, namun setiap penerbangan yang terdaftar membuat berhenti mengisi bahan bakar penasaran di Senegal dekat Tropic of Kanker di belahan Bumi Utara lebih dulu!Ketika dipetakan di Bumi yang datar alasan mengapa jelas untuk dilihat, bagaimanapun, Senegal sebenarnya langsung berada di garis lurus setengah jalan di antara keduanya. ”

Iklim, Cuaca & Suhu (49-55)

49 “Jika Bumi adalah bola pemintalan yang dipanaskan oleh Matahari sejauh 93 juta mil, tidak mungkin untuk mengalami musim panas yang terik di musim panas sementara di Afrika hanya beberapa ribu mil jauhnya dari musim dingin yang membeku. Kutub Arktik / Antartika mengalami sedikit atau tidak ada panas dari Bumi. Sun sama sekali.Jika panas dari Matahari menempuh jarak 93.000.000 mil ke gurun Sahara, tidak masuk akal untuk menyatakan bahwa sekitar 4.000 mil (0,00004%) lebih jauh ke Antartika akan sepenuhnya meniadakan panas terik yang mengakibatkan perbedaan drastis seperti itu. ”

50Jika Bumi benar-benar bola dunia, kutub Arktik dan Antartika dan daerah garis lintang yang sebanding Utara dan Selatan dari khatulistiwa harus berbagi kondisi dan karakteristik yang sama seperti suhu yang sebanding, perubahan musim, panjang siang hari, tanaman dan kehidupan binatang.Namun dalam kenyataannya, wilayah Arktik / Antartika dan wilayah garis lintang Utara / Selatan yang sama sangat berbeda dalam banyak hal yang sepenuhnya tidak konsisten dengan model bola dan sepenuhnya konsisten dengan model datar. ”

51Antartika sejauh ini merupakan tempat terdingin di Bumi dengan suhu tahunan rata-rata sekitar -57 derajat Fahrenheit, dan rekor terendah -135,8! Suhu tahunan rata-rata di Kutub Utara, bagaimanapun, adalah 4 derajat yang relatif hangat. Sepanjang tahun, suhu di Antartika bervariasi kurang dari setengah jumlah pada garis lintang Arctic yang sebanding.Wilayah Arktik Utara menikmati musim panas yang cukup hangat dan musim dingin yang dapat dikelola, sedangkan wilayah Antartika Selatan tidak pernah cukup hangat untuk mencairkan salju abadi dan es. Pada putaran, goyangan, bola Bumi berputar secara seragam di sekitar Matahari, suhu dan musim Arktik dan Antartika tidak boleh sangat bervariasi. ”

52 “Islandia di 65 derajat Lintang Utara adalah rumah bagi 870 spesies tanaman asli dan beragam kehidupan binatang.Bandingkan ini dengan Isle of Georgia di hanya 54 derajat Lintang Selatan di mana hanya ada 18 spesies tanaman asli dan kehidupan binatang hampir tidak ada.Garis lintang yang sama dengan Kanada atau Inggris di Utara di mana hutan lebat dari berbagai pohon tinggi berlimpah, Kapten Cook terkenal menulis tentang Georgia bahwa ia tidak dapat menemukan semak tunggal yang cukup besar untuk membuat tusuk gigi!Cook menulis, “Tidak ada pohon yang harus dilihat.Tanah yang terletak di sebelah selatan ditakdirkan oleh alam hingga kebekuan abadi – tidak pernah merasakan hangatnya sinar matahari;yang aspek mengerikan dan biadabnya saya tidak memiliki kata-kata untuk menggambarkan.Bahkan kehidupan laut sangat jarang di bidang tertentu, dan burung laut jarang terlihat terbang di atas sampah yang sepi itu.Perbedaan antara batas kehidupan organik di zona Arktik dan Antartika sangat luar biasa dan signifikan. ”

53Di tempat-tempat dengan garis lintang Utara dan Selatan yang sebanding, Matahari berperilaku sangat berbeda daripada di Bumi yang berputar, tetapi bagaimana seharusnya di Bumi yang datar. Sebagai contoh, hari-hari musim panas terpanjang di Utara khatulistiwa jauh lebih panjang daripada di Selatan khatulistiwa, dan hari-hari musim dingin terpendek Utara khatulistiwa jauh lebih pendek daripada yang terpendek di Selatan khatulistiwa. Hal ini tidak dapat dijelaskan pada bola Bumi yang berputar secara seragam dan bergoyang-goyang tetapi tepat pada model datar dengan Matahari bergerak melingkar di atas dan di sekitar Bumi dari Tropic ke Tropic. ”

  • Salah. Di tempat-tempat lintang yang mirip, Matahari berperilaku sama pada posisi Matahari yang sama.

54Di tempat-tempat yang sebanding dengan garis lintang Utara dan Selatan, fajar dan senja terjadi sangat berbeda dari yang terjadi pada bola yang berputar, tetapi tepatnya bagaimana mereka seharusnya berada di Bumi yang datar. Di fajar Utara dan senja datang perlahan dan bertahan jauh lebih lama daripada di Selatan di mana mereka datang dan pergi dengan sangat cepat. Tempat-tempat tertentu di Utara senja dapat berlangsung selama lebih dari satu jam sementara di lintang Selatan yang sebanding dalam beberapa menit sinar matahari benar-benar menghilang. Hal ini tidak dapat dijelaskan pada bola Bumi yang berputar secara seragam, tetapi persis seperti yang diharapkan pada Bumi yang datar dengan Matahari bergerak lebih cepat, lingkaran yang lebih luas di Selatan, dan lingkaran yang lebih sempit dan sempit di Utara.”

  • Salah. Di tempat-tempat dengan garis lintang yang sebanding, fajar dan senja terjadi persis sama pada siklus Matahari yang sebanding.

55Jika Matahari berputar-putar di atas Bumi setiap 24 jam, mantap bergerak dari Tropic ke Tropic setiap 6 bulan, maka daerah Utara, tengah setiap tahun akan menerima jauh lebih banyak panas dan sinar matahari daripada wilayah circumferential Selatan. Karena Matahari harus menyapu kawasan Selatan yang lebih besar dalam 24 jam yang sama, maka harus melewati wilayah utara yang lebih kecil, jalurnya harus secara proporsional juga lebih cepat. Ini dengan sempurna menjelaskan perbedaan suhu Arktik / Antartika, musim, panjang siang hari, tanaman dan kehidupan binatang; inilah sebabnya mengapa fajar subuh di Antartika dan senja sore sangat mendadak dibandingkan dengan Utara; dan ini menjelaskan mengapa banyak malam musim dingin Arktik Matahari tidak terbenam sama sekali! ”

  • Salah. Di tempat-tempat dengan garis lintang yang sebanding, di Utara dan Selatan, mereka menerima jumlah sinar matahari yang sama pada siklus Matahari yang sama.

Matahari Tengah Malam / Midnight Sun (56-59)

56Matahari Tengah Malam” adalah fenomena Arktik yang terjadi setiap tahun selama titik balik matahari musim panas di mana selama beberapa hari lurus seorang pengamat yang secara signifikan cukup jauh di utara dapat menyaksikan matahari mengelilingi lingkaran di atas kepala, naik dan turun di langit sepanjang hari, tetapi tidak pernah pengaturan sepenuhnya untuk lebih dari 72 jam! Jika Bumi sebenarnya adalah dunia yang berputar berputar mengelilingi Matahari, satu-satunya tempat fenomena seperti Matahari Tengah Malam dapat diamati akan berada di kutub. Sudut pandang lainnya dari 89 derajat lintang ke bawah tidak akan pernah, terlepas dari kemiringan atau kemiringan apa pun, lihat Matahari selama 24 jam penuh. Untuk melihat Matahari untuk keseluruhan revolusi pada bola dunia yang berputar di titik selain kutub, Anda harus melihat melalui mil dan mil tanah dan laut untuk bagian dari revolusi! ”

57Pendirian mengklaim Sun Midnight IS berpengalaman di Antartika tetapi mereka tidak memiliki video yang tidak menunjukkan hal ini, mereka juga tidak mengizinkan penjelajah independen untuk melakukan perjalanan ke Antartika selama titik balik matahari musim dingin untuk memverifikasi atau menyangkal klaim ini. Sebaliknya, ada lusinan video yang tidak dipamerkan yang tersedia untuk umum menunjukkan Arktik Tengah Malam Matahari dan telah diverifikasi tanpa keraguan. ”

58Masyarakat Geografis Kerajaan Belgia di“ Ekspedisi Antartika Belge ”mereka, mencatat bahwa selama bagian paling parah dari musim dingin Antartika, dari 71 derajat Lintang Selatan dan seterusnya, matahari terbenam pada tanggal 17 Mei dan tidak terlihat di atas cakrawala lagi hingga Juli. 21! Ini benar-benar bertentangan dengan teori bola-Bumi, tetapi mudah dijelaskan oleh model Bumi-datar. Matahari Tengah Malam terlihat dari ketinggian tinggi di lintang Utara yang ekstrem selama musim panas Arktik karena Matahari, pada siklus paling dalam, berputar cukup dekat di sekitar pusat kutub yang tetap terlihat di atas cakrawala bagi seseorang di titik yang menguntungkan.Demikian pula, di garis lintang Selatan yang ekstrem selama musim panas Arktik, Matahari benar-benar menghilang dari pandangan selama lebih dari 2 bulan karena ada di Northern Tropic, pada busur paling dalam dari perjalanan bumerangnya, Matahari berputar-putar di pusat Utara terlalu ketat untuk dilihat. dari lingkar Selatan. “

  • 71° Selatan berada di selatan Lingkar Antartika (~ 66,5 °) dan dengan demikian akan mengalami Matahari tengah malam pada suatu saat dalam setahun. Ini sesuai dengan ekspektasi pada model Bumi bulat.

59Mengutip Gabrielle Henriet,” Teori rotasi bumi mungkin sekali dan untuk semua pasti dibuang sebagai tidak praktis dengan menunjukkan ketidaksinkronan berikut. Dikatakan bahwa rotasi membutuhkan waktu dua puluh empat jam dan kecepatannya seragam, dalam hal ini, tentu saja, siang dan malam harus memiliki durasi identik dua belas jam setiap sepanjang tahun.Matahari seharusnya selalu naik di pagi hari dan terbenam di malam hari pada jam yang sama, dengan hasil bahwa itu akan menjadi ekuinoks setiap hari dari tanggal 1 Januari hingga 31 Desember. Seseorang harus berhenti dan merenungkan hal ini sebelum mengatakan bahwa bumi memiliki pergerakan rotasi.Bagaimana sistem gravitasi memperhitungkan variasi musiman dalam durasi siang dan malam jika bumi berputar dengan kecepatan seragam dalam dua puluh empat jam !? ”

  • Dia tidak memperhitungkan adanya kemiringan sumbu rotasi Bumi.

Lengkungan Bumi (60-80)

60 “Siapa pun dapat membuktikan cakrawala laut lurus sempurna dan seluruh Bumi datar sempurna menggunakan tidak lebih dari tingkat, tripod dan papan kayu.Pada ketinggian apa pun di atas permukaan laut, cukup perbaiki papan yang panjangnya 6-12 kaki, halus, dan diratakan pada tripod dan amati cakrawala dari ketinggian mata di belakangnya.Cakrawala yang jauh akan selalu sejajar sejajar dengan tepi atas papan.Lebih jauh lagi, jika Anda bergerak dalam setengah lingkaran dari satu ujung papan ke ujung lainnya sambil mengamati cakrawala di atas tepi atas, Anda akan dapat menelusuri dataran yang datar dan datar 10-20 mil tergantung pada ketinggian Anda.Ini tidak mungkin jika Bumi adalah lingkaran dunia 25.000 mil;cakrawala akan sejajar di atas tengah papan tetapi kemudian secara bertahap, terasa menurun ke ekstremitas.Hanya sepuluh mil di setiap sisi akan membutuhkan kelengkungan yang mudah terlihat dari 66,6 kaki dari masing-masing ujung ke pusat. ”

61“Jika Bumi benar-benar bola besar sepanjang 25.000 mil, cakrawala akan terlihat melengkung bahkan di permukaan laut, dan segala sesuatu di atau mendekati cakrawala akan tampak miring ke belakang sedikit dari perspektif Anda.Bangunan yang jauh di sepanjang cakrawala akan terlihat seperti menara miring Piza yang jatuh menjauh dari pengamat. Balon udara yang lepas landas kemudian hanyut terus menjauh dari Anda, di atas bola Bumi akan perlahan dan terus-menerus tampak bersandar lebih jauh dan semakin jauh terbang, bagian bawah keranjang datang secara bertahap ke dalam pandangan sebagai bagian atas balon menghilang dari pandangan. Namun dalam kenyataannya, bangunan, balon, pohon, orang, apa saja dan segala sesuatu dengan sudut kanan ke tanah / cakrawala tetap demikian terlepas dari jarak atau ketinggian pengamat. ”

62Eksperimen Samuel Rowbotham di Old Bedford Level membuktikan secara konklusif saluran air menjadi benar-benar datar selama 6 mil peregangan. Mula-mula dia berdiri di kanal dengan teleskopnya memegang 8 inci di atas permukaan air, kemudian temannya di kapal dengan bendera 5 kaki tinggi berlayar sejauh 6 mil. Jika Bumi adalah bola 25.000 mil dalam keliling, 6 mil hamparan air seharusnya terdiri dari busur tepat 6 kaki di tengah, sehingga seluruh kapal dan bendera seharusnya telah hilang, padahal sebenarnya seluruh perahu dan bendera tetap terlihat di ketinggian yang sama untuk seluruh perjalanan. “

63Dalam percobaan kedua, Dr. Rowbotham menempelkan bendera setinggi 5 kaki di sepanjang garis pantai, satu di setiap mil penanda. Kemudian menggunakan teleskopnya yang dipasang pada 5 kaki tepat di belakang bendera pertama yang melihat puncak semua 6 bendera yang berbaris dalam garis lurus sempurna. Jika Bumi adalah bola 25.000 mil dalam keliling bendera harus semakin menurun setelah garis pandang pertama, kedua akan turun 8 inci, 32 inci untuk ketiga, 6 kaki untuk keempat, 10 kaki 8 inci untuk kelima, dan 16 kaki 8 inci untuk yang keenam. “

64“Mengutip” Bumi Bukan Globe! “Oleh Samuel Rowbotham,” Telah diketahui bahwa cakrawala di laut, jarak apa pun yang mungkin meluas ke kanan dan kiri pengamat di darat, selalu tampak sebagai garis lurus. Percobaan berikut telah dicoba di berbagai bagian negara. Di Brighton, di sebuah dataran tinggi di dekat arena balap, dua tiang dipasang di bumi sejauh enam yard, dan tepat di seberang lautan. Di antara kutub-kutub ini sebuah garis lurus membentang sejajar dengan cakrawala. Dari pusat garis, pemandangan yang dianut tidak kurang dari 20 mil di setiap sisi membuat jarak 40 mil. Sebuah kapal diamati berlayar langsung ke arah barat; garis memotong tali-temali sedikit di atas bulwark, yang dilakukannya selama beberapa jam atau sampai kapal itu mengarungi seluruh jarak 40 mil. Kapal yang muncul dari arah timur harus naik ke pesawat yang miring sejauh 20 mil sampai tiba di pusat busur, dari mana kapal itu harus turun untuk jarak yang sama. Alun-alun 20 mil dikalikan 8 inci memberikan 266 kaki sebagai jumlah kapal akan berada di bawah garis di awal dan di ujung 40 mil. “

65Juga Mengutip Dr. Rowbotham,” Di pantai dekat Waterloo, beberapa mil di utara Liverpool, teleskop yang bagus diperbaiki, pada ketinggian 6 kaki di atas permukaan air. Itu diarahkan ke kapal uap besar, hanya meninggalkan Sungai Mersey, dan berlayar ke Dublin. Berangsur-angsur kepala tiang kapal yang semakin surut mendekati cakrawala, sampai, akhirnya, setelah lebih dari empat jam berlalu, menghilang. Laju biasa pelayaran kapal uap Dublin adalah delapan mil per jam; sehingga kapal akan, setidaknya, tiga puluh dua mil jauh ketika kepala tiang datang ke cakrawala. Ketinggian 6 kaki teleskop akan membutuhkan tiga mil untuk dikurangi untuk konveksitas, yang akan meninggalkan dua puluh sembilan mil, persegi yang, dikalikan dengan 8 inci, memberikan 560 kaki; dikurangi 80 kaki untuk ketinggian tiang utama, dan kami menemukan bahwa, menurut doktrin kekeruhan, tiang-kepala kapal uap yang menghadap ke luar seharusnya 480 kaki di bawah cakrawala.Banyak eksperimen lain semacam ini telah dilakukan pada kapal uap laut, dan selalu dengan hasil yang sepenuhnya bertentangan dengan teori bahwa bumi adalah bola dunia. ”

66Dr. Rowbotham melakukan beberapa eksperimen lain menggunakan teleskop, level spirit, sextant dan “theodolites,” instrumen presisi khusus yang digunakan untuk mengukur sudut dalam bidang horizontal atau vertikal. Dengan memposisikan mereka pada ketinggian yang sama yang ditujukan satu sama lain secara berturut-turut, ia terbukti berulang kali di atas Bumi menjadi datar sempurna untuk mil tanpa satu inci kelengkungan. Penemuannya menyebabkan kehebohan dalam komunitas ilmiah dan berkat 30 tahun usahanya, bentuk Bumi menjadi topik hangat perdebatan di sekitar pergantian abad kesembilan belas. ”

67Jarak di Laut Irlandia dari Pelabuhan Douglas Isle of Man ke Great Orm’s Head di North Wales adalah 60 mil. Jika Bumi adalah bola dunia maka permukaan air di antara mereka akan membentuk busur sepanjang 60 mil, pusat yang menjulang setinggi 1.944 kaki dari garis pantai di kedua ujungnya. Ini terkenal dan mudah diverifikasi, namun, pada hari yang cerah, dari ketinggian sederhana 100 kaki, Kepala Orm Agung terlihat dari Pelabuhan Douglas. Ini benar-benar mustahil di dunia seluas 25.000 mil. Dengan asumsi ketinggian 100 kaki menyebabkan cakrawala muncul sekitar 13 mil, 47 mil yang tersisa berarti garis pantai Welsh masih harus jatuh mustahil 1472 kaki di bawah garis pandang! ”

68Garis langit Philadelphia terlihat jelas dari Apple Pie Hill di New Jersey Pine Barrens, 40 mil jauhnya. Jika Bumi adalah bola 25.000 mil di lingkar, memfaktorkan di ketinggian 205 kaki Apple Pie Hill, cakrawala Philly harus tetap tersembunyi di balik 335 kaki kelengkungan. ”

69Cakrawala Kota New York jelas terlihat dari Bear Mountain Harriman State Park yang berjarak 60 mil. Jika Bumi adalah bola 25.000 mil dalam keliling, melihat dari puncak gunung 1.228 kaki Bear Mountain, Teorema Pythagoras menentukan jarak ke cakrawala menjadi 1,23 kali akar kuadrat ketinggian di kaki, langit NYC harus tidak terlihat di belakang 170 kaki bumi melengkung . “

70Dari Washington’s Rock di New Jersey, hanya dengan ketinggian 400 kaki, mungkin pada hari yang cerah untuk melihat langit langit New York dan Philadelphia dalam arah yang berlawanan pada saat yang sama yang mencakup jarak total 120 mil! Jika Bumi adalah bola sepanjang 25.000 mil, kedua skylines ini harus tersembunyi di balik lebih dari 800 kaki kelengkungan Bumi. ”

71Sering kali kita bisa melihat cakrawala Chicago dari permukaan laut sejauh 60 mil di seberang Danau Michigan. Pada tahun 2015 setelah fotografer Joshua Nowicki memotret fenomena ini, beberapa saluran berita dengan cepat mengklaim fotonya sebagai “fatamorgana superior”, sebuah anomali atmosfer yang disebabkan oleh inversi suhu. Sementara ini memang terjadi, cakrawala yang dimaksud menghadap ke kanan ke atas dan jelas terlihat tidak seperti khayalan ilusi yang kabur, dan pada bola-Bumi keliling 25.000 mil harus 2.400 kaki di bawah cakrawala. “

7216 Oktober 1854 surat kabar Times melaporkan kunjungan Ratu ke Great Grimsby dari rekaman Hull mereka dapat melihat menara dermaga setinggi 300 kaki dari jarak 70 mil. Pada bola-Bumi 25.000 mil di lingkar, memfaktorkan elevasi 10 kaki mereka di atas air dan tinggi menara 300 kaki, pada 70 mil jauhnya menara dermaga seharusnya tetap keseluruhan 2.600 kaki di bawah cakrawala. “

73Pada tahun 1872, Kapten Gibson dan awak kapal, yang mengarungi kapal“ Thomas Wood ”dari Tiongkok ke London, melaporkan melihat keseluruhan Pulau St. Helena pada hari yang cerah dari jarak 75 mil. Anjak mereka selama pengukuran pada bola-Bumi 25.000 mil di lingkar, ditemukan pulau seharusnya 3,650 kaki di bawah garis pandang mereka. “

74Dari Genoa, Italia pada ketinggian hanya 70 kaki di atas permukaan laut, pulau Gorgona sering dapat dilihat 81 mil jauhnya. Jika Bumi adalah bola 25.000 mil di lingkar, Gorgona harus tersembunyi di balik 3,332 kaki kelengkungan. ”

75Dari Genoa, Italia pada ketinggian hanya 70 kaki di atas permukaan laut, pulau Corsica sering dapat dilihat sejauh 99 mil. Jika Bumi memiliki lingkaran 25.000 mil, Corsica harus jatuh 5,245 kaki, hampir satu mil di bawah cakrawala. ”

76Dari Genoa, Italia, 70 meter di atas permukaan laut, pulau Capraia yang berjarak 102 mil sering dapat dilihat juga. Jika Bumi adalah bola 25.000 mil di lingkar, Capraia harus selalu tetap tersembunyi di belakang 5.605 kaki, lebih dari satu mil dari kelengkungan seharusnya. “

77Juga dari Genoa, pada hari-hari cerah, pulau Elba dapat terlihat luar biasa, 125 mil jauhnya! Jika Bumi adalah bola 25.000 mil di lingkar, Elba harus selamanya tak terlihat di balik 8770 kaki kelengkungan. “

78Dari Anchorage, Alaska pada ketinggian 102 kaki, pada hari-hari yang jelas Gunung Foraker dapat dilihat dengan mata telanjang sejauh 120 mil. Jika Bumi adalah bola 25.000 mil dalam keliling, puncak 17.400 Gunung Foraker harus bersandar jauh dari pengamat yang tertutup oleh 7.719 kaki Bumi yang melengkung. Kenyataannya, bagaimanapun, seluruh gunung dapat dengan mudah dilihat berdiri langsung dari pangkal ke puncak. ”

79Dari Anchorage, Alaska pada ketinggian 102 kaki, pada hari-hari yang cerah Gunung McKinley dapat dilihat dengan mata telanjang dari 130 mil jauhnya. Jika Bumi adalah bola 25.000 mil dalam keliling, puncak Gunung 20.220 kaki McKinley harus bersandar jauh dari pengamat dan hampir setengah tertutup oleh 9.220 kaki Bumi melengkung.Kenyataannya, bagaimanapun, seluruh gunung dapat dengan mudah dilihat berdiri langsung dari pangkal ke puncak. ”

80Dalam Chambers ‘Journal, Februari 1895, seorang pelaut dekat Mauritius di Samudera Hindia melaporkan telah melihat sebuah kapal yang ternyata luar biasa sejauh 200 mil! Insiden ini menyebabkan banyak perdebatan sengit di kalangan kelautan pada saat itu, mendapatkan konfirmasi lebih lanjut di Aden, Yaman di mana saksi lain melaporkan melihat kapal uap Bombay yang hilang dari 200 mil jauhnya. Dia benar menyatakan penampilan yang tepat, lokasi dan arah kapal uap semuanya dikuatkan dan dikonfirmasi benar oleh mereka yang ada di pesawat. Penampakan seperti itu benar-benar tidak dapat dijelaskan jika Bumi sebenarnya adalah bola 25.000 mil di sekitar, karena kapal yang berjarak 200 mil jauhnya akan jatuh sekitar 5 mil di bawah garis pandang! ”

  • Ini adalah sebuah hikayat. Tidak mungkin untuk mengidentifikasi kapal dari 200 mil jauhnya, bahkan menggunakan teknologi saat ini. Dan tidak mungkin memastikan jaraknya sejauh 200 mil.

Mercusuar (81-93)

81Jarak dari mana berbagai lampu mercusuar di seluruh dunia terlihat di laut jauh melebihi apa yang dapat ditemukan pada bola-Bumi keliling 25.000 mil. Misalnya, Cahaya Dunkerque di Prancis selatan pada ketinggian 194 kaki terlihat dari perahu (10 kaki di atas permukaan laut) 28 mil jauhnya.Spherical trigonometry menentukan bahwa jika Bumi adalah bola dunia dengan kelengkungan yang diberikan 8 inci per mil kuadrat, cahaya ini harus disembunyikan 190 kaki di bawah cakrawala. ”

82 “The Port Nicholson Light di Selandia Baru adalah 420 meter di atas permukaan laut dan terlihat dari 35 mil jauhnya di mana seharusnya 220 kaki di bawah cakrawala.”

83 “Cahaya Egerö di Norwegia adalah 154 meter di atas air yang tinggi dan terlihat dari 28 kilometer undang-undang di mana seharusnya 230 kaki di bawah cakrawala.”

84 “Cahaya di Madras, di Esplanade, tingginya 132 kaki dan terlihat dari 28 mil jauhnya, di mana seharusnya 250 kaki di bawah garis pandang.”

85 “The Cordonan Light di pantai barat Perancis adalah 207 meter dan terlihat dari 31 mil jauhnya, di mana itu harus 280 kaki di bawah garis pandang.”

86 “Cahaya di Tanjung Bonavista, Newfoundland adalah 150 meter di atas permukaan laut dan terlihat di 35 mil, di mana seharusnya 491 kaki di bawah cakrawala.”

87 “Mercusuar mercusuar dari Gereja Paroki St. Botolphp di Boston adalah 290 meter dan terlihat dari lebih dari 40 mil jauhnya, di mana itu harus disembunyikan 800 kaki penuh di bawah cakrawala!”

88 “Mercusuar Isle of Wight di Inggris adalah 180 kaki tinggi dan dapat dilihat hingga 42 mil jauhnya, jarak di mana para astronom modern mengatakan cahaya harus jatuh 996 kaki di bawah garis pandang.”

89 “Mercusuar Cape L’Agulhas di Afrika Selatan adalah 33 kaki tingginya, 238 meter di atas permukaan laut, dan dapat dilihat lebih dari 50 mil. Jika dunia adalah bola dunia, cahaya ini akan jatuh 1.400 meter di bawah garis pandang pengamat. ”

90 “Patung Liberty di New York berdiri 326 meter di atas permukaan laut dan pada hari yang cerah dapat dilihat sejauh 60 mil jauhnya. Jika Bumi adalah bola dunia, itu akan menempatkan Lady Liberty pada ketinggian 2.074 kaki di bawah cakrawala. ”

91 “Mercusuar di Port Said, Mesir, pada ketinggian hanya 60 kaki telah terlihat 58 mil yang menakjubkan, di mana, menurut astronomi modern itu harus 2.182 kaki di bawah garis pandang!”

92 “Puncak menara Notre Dame Antwerp berdiri 403 kaki dari kaki menara dengan Strasburg berukuran 468 kaki di atas permukaan laut. Dengan bantuan teleskop, kapal dapat dibedakan di cakrawala dan kapten menyatakan bahwa mereka dapat melihat menara katedral dari jarak 150 mil yang menakjubkan. Jika Bumi adalah bola dunia, bagaimanapun, pada jarak itu puncak menara harusnya satu mil penuh, 5.280 kaki di bawah cakrawala! ”

93 “Saluran St. George di antara Holyhead dan Kingstown Harbor dekat Dublin adalah 60 mil. Ketika setengah jalan melintasi penumpang feri akan melihat di belakang mereka cahaya di dermaga Holyhead serta di depan mereka cahaya Poolbeg di Dublin Bay. Lampu Holyhead Pier adalah 44 kaki tinggi, sedangkan mercusuar Poolbeg 68 kaki, oleh karena itu kapal di tengah saluran, 30 mil dari kedua sisi berdiri di dek 24 meter di atas air, dapat dengan jelas melihat kedua lampu. Pada bola Bumi 25.000 mil di lingkar, bagaimanapun, kedua lampu harus disembunyikan di bawah kedua cakrawala dengan lebih dari 300 kaki!

Pengamatan Objek di Kejauhan (94-96)

94 “Dari dataran tinggi dekat Pelabuhan Portsmouth di Hampshire, Inggris mencari di seberang Spithead ke Isle of Wight, seluruh pangkalan pulau itu, di mana air dan tanah bersatu membentuk garis lurus yang sempurna 22 undang-undang bermil-mil panjang. Menurut teori bola-Bumi, Isle of Wight harus menurun 80 kaki dari pusat di setiap sisi untuk memperhitungkan kelengkungan yang diperlukan. Namun, cross-hair of aodolite yang baik yang diarahkan ke sana, telah berulang kali menunjukkan garis tanah dan air menjadi sangat rata. ”

95 “Pada suatu hari yang cerah dari dataran tinggi dekat Pelabuhan Douglas di Isle of Man, seluruh panjang pantai North Wales sering terlihat jelas dengan mata telanjang. Dari Point of Ayr di mulut Sungai Dee ke Holyhead terdiri dari 50 mil peregangan yang juga telah berulang kali ditemukan menjadi horizontal sempurna. Jika Bumi benar-benar memiliki kelengkungan 8 inci per mil kuadrat, seperti NASA dan klaim astronomi modern, 50 mil panjang pantai Welsh terlihat di sepanjang cakrawala di Teluk Liverpool harus menurun dari titik pusat yang mudah terdeteksi 416 kaki pada setiap sisi!”

96 “Dari“ 100 Proofs the Earth is Not a Globe ”oleh William Carpenter,“ Jika kita melakukan perjalanan menyusuri Teluk Chesapeake, di malam hari, kita akan melihat ‘cahaya’ yang dipamerkan di Sharpe’s Island selama satu jam sebelum kapal uap mencapai saya t. Kita dapat mengambil posisi di dek sehingga rel dari sisi kapal akan sejajar dengan ‘cahaya’ dan di garis pandang; dan kita akan menemukan bahwa di sepanjang perjalanan cahaya itu tidak akan bervariasi sedikit pun dalam ketinggiannya yang tampak. Tapi, katakanlah bahwa jarak tiga belas mil telah dilalui, teori astronom tentang ‘kelengkungan’ menuntut perbedaan (satu atau lain cara!) Pada ketinggian cahaya yang tampak, dari 112 kaki 8 inci! Karena, bagaimanapun, tidak ada perbedaan 100 lebar rambut, kami memiliki bukti nyata bahwa air Teluk Chesapeake tidak melengkung, yang merupakan bukti bahwa Bumi bukanlah bola dunia. ”

Astronomi (97-105)

97 “NASA dan astronomi modern mengatakan Bumi adalah bola raksasa yang miring ke belakang, bergoyang-goyang dan berputar 1.000 mph di sekitar poros pusatnya, menempuh 67.000 mph mengelilingi Matahari, berputar 500.000 mph di sekitar Bima Sakti, sementara seluruh galaksi meroket 670.000.000 mph melalui Alam Semesta, dengan semua gerakan yang berasal dari ledakan kosmogenik “Big Bang” 14 miliar tahun yang lalu. Itu adalah jumlah total 670.568.000 mph di beberapa arah yang berbeda, kita semua seharusnya melaju bersama-sama, namun tidak ada yang pernah melihat, merasakan, mendengar, mengukur atau membuktikan satu gerakan ini ada sama sekali. ”

98 “NASA dan astronomi modern mengatakan Polaris, bintang Kutub Utara, berada di antara 323-434 tahun cahaya, atau sekitar 2 quadriliun mil, jauh dari kita! Pertama-tama, perhatikan bahwa antara 1.938.000.000.000 – 2.604.000.000.000.000, 000 membuat perbedaan 666.000.000.000.000 (lebih dari enam ratus triliun) mil! Jika astronomi modern bahkan tidak dapat menyepakati jarak ke bintang-bintang dalam radius ratusan trilyunan mil, mungkin “ilmu” mereka cacat dan teori mereka perlu diteliti ulang. Namun, bahkan memberikan mereka bintang-bintang mereka yang tidak jelas, tidak mungkin bagi para heliocentris untuk menjelaskan bagaimana Polaris mengelola untuk selalu tetap selaras lurus di atas Kutub Utara di seluruh Bumi yang diduga berbagai gerakan miring, bergoyang-goyang, berputar dan berputar. ”

99 “Dilihat dari bola-Bumi, Polaris, yang terletak langsung di Kutub Utara, seharusnya tidak terlihat di mana saja di belahan bumi Selatan. Untuk Polaris dilihat dari belahan Bumi Selatan Bumi yang bulat, pengamat harus entah bagaimana mencari “melalui bola dunia”, dan mil daratan dan lautan harus transparan. Polaris dapat dilihat, bagaimanapun, hingga lebih dari 20 derajat Lintang Selatan. ”

100“Jika Bumi adalah sebuah bola, Salib Selatan dan rasi bintang Selatan lainnya akan terlihat pada waktu yang sama dari setiap garis bujur pada garis lintang yang sama seperti halnya di Utara dengan Polaris dan rasi bintang di sekitarnya. Ursa Major / Minor dan banyak lainnya dapat dilihat dari setiap meridian Utara secara bersamaan sedangkan di Selatan, rasi bintang seperti Southern Cross tidak bisa. Ini membuktikan bahwa belahan Bumi Selatan tidak “terbalik” seperti pada model bola Bumi, tetapi hanya membentang lebih jauh ke luar dari titik pusat Utara seperti pada model Bumi datar. ”

101 “Sigma Octantis diklaim sebagai bintang kutub tengah Selatan yang mirip dengan Polaris, di mana bintang belahan Bumi Selatan semua berputar di sekitar arah yang berlawanan. Tidak seperti Polaris, bagaimanapun, Sigma Octantis TIDAK dapat dilihat secara bersamaan dari setiap titik di sepanjang garis lintang yang sama, itu TIDAK pusat tetapi diduga 1 derajat di luar pusat, itu TIDAK tidak bergerak, dan pada kenyataannya tidak dapat dilihat sama sekali menggunakan teleskop tersedia untuk umum! Ada spekulasi yang sah mengenai apakah Sigma Octantis ada. Either way, arah di mana bintang bergerak di atas didasarkan pada perspektif dan arah yang tepat yang Anda hadapi, bukan belahan mana Anda berada. ”

102 “Beberapa heliocentris telah mencoba untuk menunjukkan bahwa penurunan gradasi bertahap dari Bintang Pole sebagai pengamat berjalan ke arah selatan adalah bukti dari Bumi yang bulat. Jauh dari itu, deklinasi Bintang Kutub atau objek lain hanyalah hasil dari Hukum Perspektif pada permukaan datar (datar). The Law of Perspective menyatakan bahwa sudut dan ketinggian di mana suatu objek terlihat berkurang semakin jauh satu surut dari objek, sampai pada titik tertentu garis pandang dan permukaan bumi yang tampaknya bertumburan menyatu dengan titik hilang (yaitu garis horizon) di luar yang objeknya tidak terlihat. Dalam bola-Bumi model cakrawala diklaim sebagai kelengkungan Bumi, sedangkan dalam kenyataannya, cakrawala dikenal hanya garis menghilang perspektif berdasarkan kekuatan mata Anda, instrumen, cuaca dan ketinggian. ”

103 “Ada beberapa rasi bintang yang dapat dilihat dari jarak yang jauh lebih besar di atas permukaan Bumi daripada yang mungkin terjadi jika dunia adalah bola yang berputar, berputar, dan bergoyang-goyang. Misalnya, Ursa Major, sangat dekat dengan Polaris, dapat dilihat dari 90 derajat Lintang Utara (Kutub Utara) hingga ke 30 derajat Lintang Selatan. Agar hal ini bisa terjadi pada bola Bumi, para pengamat Selatan harus melihat melalui ratusan atau ribuan mil Bumi yang membengkak ke langit Utara. ”

  • Bintang Biduk atau Big Dipper
  • Sebuah bintang terlihat di beberapa titik di lokasi tertentu jika deklinasinya berada dalam garis lintang pengamat plus & minus 90 °.
  • Ursa Major adalah rasi besar yang mencakup deklinasi + 30 ° hingga + 73 °. Seorang pengamat pada lintang -30 ° akan dapat melihat bintang mana pun yang mengalami deklinasi hingga + 60 °, pada suatu saat dalam setahun. Ini memungkinkan melihat sebagian besar rasi Ursa Major.
  • Menentukan Kenampakan Bintang Dari Deklinasinya dan Lintang Pengamat
  • Semuanya konsisten dengan model Bumi bulat.

104Konstelasi Vulpecula dapat dilihat dari 90 derajat Lintang Utara, sepanjang jalan sampai 55 derajat Lintang Selatan. Taurus, Pisces dan Leo dapat dilihat dari 90 derajat Utara sampai ke 65 derajat Selatan. Seorang pengamat pada bola-Bumi, terlepas dari kemiringan atau kemiringan apa pun, tidak seharusnya secara logis dapat melihat sejauh ini. ”

  • Sebuah bintang terlihat di lokasi tertentu jika deklinasinya antara derajat lintang pengamat plus & minus 90°.
  • Vulpecula memiliki deklinasi antara + 20 ° & + 30 °.
  • Taurus memiliki deklinasi antara -2 ° & + 31 °.
  • Pisces memiliki deklinasi antara -6 ° & 33 °.
  • Leo memiliki deklinasi antara -7 ° & 33 °
  • Seorang pengamat pada -55 ° akan dapat melihat bintang hingga +35°, termasuk semua Vulpecula.
  • Seorang pengamat di -65° akan dapat melihat bintang hingga +25°, yang mencakup sebagian besar Taurus, Pisces, dan Leo.
  • Menentukan Kenampakan Bintang Dari Deklinasinya dan Lintang Pengamat
  • Semuanya konsisten dengan model Bumi bulat.

105 “Aquarius dan Libra dapat dilihat dari 65 derajat Utara hingga 90 derajat Selatan! Konstelasi Virgo terlihat dari 80 derajat Utara hingga 80 derajat Selatan, dan Orion dapat dilihat dari 85 derajat Utara sampai ke 75 derajat Lintang Selatan! Ini semua hanya mungkin karena “belahan” tidak berada di semua bidang tetapi lingkaran lingkaran konsentris memanjang keluar dari Kutub Utara bagian utara dengan bintang-bintang berotasi di atas dan di sekitar. “

  • Sebuah bintang terlihat di lokasi tertentu jika deklinasinya antara derajat lintang pengamat plus & minus 90°.
  • Aquarius memiliki deklinasi antara -24 ° & + 3 °
  • Libra memiliki deklinasi antara -30 ° & + 1 °
  • Virgo memiliki deklinasi antara -23 ° & + 15 °
  • Orion memiliki deklinasi antara -11 ° & 23 °
  • Seorang pengamat di +65° dapat melihat bintang hingga -25°, yang mencakup semua Aquarius dan sebagian besar Libra.
  • Seorang pengamat di +80° dapat melihat bintang hingga -10°, yang mencakup sebagian besar Virgo.
  • Seorang pengamat di -80° dapat melihat bintang hingga +10°, yang mencakup sebagian besar Virgo.
  • Seorang pengamat +85° dapat melihat bintang hingga -5°, yang mencakup sebagian besar Orion.
  • Seorang pengamat -75° dapat melihat bintang hingga +15°, yang mencakup sebagian besar Orion.
  • Menentukan Kenampakan Bintang Dari Deklinasinya dan Lintang Pengamat
  • Semuanya konsisten dengan model Bumi bulat.

Kutub, Kompas & Magnet Bumi Bumi (106-108)

106“Apa yang disebut“ Kutub Selatan ”hanyalah titik sembarang di sepanjang es Antartika yang ditandai dengan tiang pangkas merah dan putih yang diatapi bola logam-Bumi. Tiang Selatan seremonial ini memang diakui dan terbukti BUKAN Kutub Selatan yang sebenarnya, namun, karena Kutub Selatan yang sebenarnya dapat dibuktikan dengan bantuan kompas yang menunjukkan Utara 360 derajat di sekitar pengamat. Karena prestasi ini tidak pernah tercapai, model tetap teori murni, bersama dengan alasan pendirian bahwa kutub geomagnetik seharusnya terus bergerak membuat verifikasi klaim mereka mustahil. ”

  • Kutub geografis selatan berbeda dari kutub magnetik selatan. Hal yang sama terjadi dengan kutub utara.
  • Dengan “penalaran” seperti itu, kita juga bisa “membuktikan” kutub utara tidak ada.

107Magnet cincin dari jenis yang ditemukan di pengeras suara memiliki kutub Utara di bagian utara dengan kutub“ Selatan ”berlawanan yang sebenarnya adalah semua titik di sepanjang lingkar luar. Ini dengan sempurna menunjukkan magnet Bumi kita yang datar, sedangkan sumber magnetisme yang diduga dalam bola-model Bumi dipancarkan dari sebuah inti magnetik hipotetis di pusat bola yang mereka klaim dengan nyaman menyebabkan kedua kutub terus bergerak sehingga menghindari verifikasi independen. di dua “seremonial poles” mereka. Kenyataannya operasi pengeboran terdalam dalam sejarah, Rusia Kola Ultradeep, berhasil hanya 8 mil ke bawah, sehingga seluruh model bola-Bumi yang diajarkan di sekolah-sekolah menunjukkan kerak, mantel luar, batin- mantel, lapisan inti-dalam dan inti-dalam semuanya murni spekulasi karena kita tidak pernah menembus menembus lapisan luar. ”

108 “Kompas mariner adalah instrumen yang mustahil dan tidak masuk akal untuk digunakan pada bola-Bumi. Ini secara bersamaan menunjukkan Utara dan Selatan di atas permukaan yang datar, namun mengklaim sebagai pin-menunjuk dua kutub geomagnetik terus bergerak di ujung berlawanan dari bola berputar yang berasal dari inti logam cair hipotetis. Jika jarum kompas benar-benar tertarik ke kutub Utara bola dunia, jarum “Selatan” yang berlawanan akan benar-benar mengarah ke atas dan ke luar angkasa. “

Perjalanan Mengelilingi Bumi (109-111)

109Tidak ada titik tetap” Timur “atau” Barat “sama seperti tidak ada” Selatan “yang tetap. Kutub pusat Utara adalah satu-satunya titik tetap yang terbukti di Bumi kita yang datar, dengan Selatan adalah semua garis lurus keluar dari kutub, Timur dan Barat menjadi lingkaran konsentris pada sudut kanan konstan 90 derajat dari kutub. Sebuah perjalanan keliling dunia dengan mengelilingi Polaris terus di sebelah kanan Anda, sementara perjalanan keliling timur berkeliling dengan Polaris selalu di sebelah kiri Anda. ”

  • Ini adalah “penjelasan” dari model Bumi-datar mereka dan tidak ada klaim yang dibuat. Tidak ada bantahan yang diperlukan.
  • Bintang Utara Polaris

110Magellan dan lain-lain ‘Lingkaran Bumi Timur / Barat sering dikutip sebagai bukti model bola.Namun, dalam kenyataannya, berlayar atau terbang pada sudut pandang ke kutub Utara dan akhirnya kembali ke lokasi asal tidak lebih sulit atau misterius daripada melakukannya di dunia. Sama seperti kompas arsitek yang dapat menempatkan titik pusatnya pada selembar kertas datar dan menelusuri lingkaran di sekitar “tiang”, demikian juga sebuah kapal atau pesawat dapat mengelilingi Bumi yang datar ”

  • Ini adalah “penjelasan” tentang bagaimana sesuatu bekerja di Bumi yang datar. Tidak ada bantahan yang diperlukan.

111Karena Kutub Utara dan Antartika tertutup es dan menjaga zona“ larangan terbang ”, tidak ada kapal atau pesawat yang pernah diketahui mengelilingi Bumi dalam arah Utara / Selatan. Satu-satunya jenis pelayaran yang tidak dapat terjadi di Bumi yang datar adalah Utara / Selatan, yang kemungkinan besar merupakan alasan untuk pembatasan penerbangan yang sangat ketat. Fakta bahwa belum ada satu pun penjelajahan Bumi Utara / Selatan yang diverifikasi yang berfungsi sebagai bukti nyata bahwa dunia bukanlah sebuah bola. ”

Lain-Lain (112-114)

112Matahari membawa siang ke setiap zona waktu saat melewati langsung di atas kepala setiap titik demarkasi 15 derajat, 24 kali per hari dalam jalur melingkar di atas dan di sekitar Bumi. Jika zona waktu bukan disebabkan oleh pemintalan yang seragam dari bola-Bumi di sekitar Matahari, setiap 6 bulan ketika Bumi menemukan dirinya di sisi berlawanan dari Matahari, jam di seluruh Bumi harus membalik 12 jam, hari akan menjadi malam dan malam akan menjadi siang. ”

113Gagasan bahwa orang berdiri, kapal berlayar dan pesawat terbang terbalik di bagian-bagian tertentu dari Bumi sementara yang lain miring pada 90 derajat dan semua sudut mustahil lainnya adalah absurditas yang lengkap. Gagasan bahwa seorang pria yang menggali lubang lurus ke bawah pada akhirnya dapat mencapai langit di sisi lain adalah hal yang menggelikan. Akal sehat mengatakan kepada setiap orang yang berpikiran bebas dengan benar bahwa ada benar-benar “naik” dan “turun” di alam, tidak seperti “semuanya relatif” retorika paradigma Newtonian / Einsteinian. “

  • Ini adalah fallacy ‘appeal to the stone’: menolak argumen tidak masuk akal tanpa memberitahukan dimana tidak masuk akalnya.
  • Arah Atas dan Bawah
  • TBD(appeal-to-stone)

114Mengutip,” Tentang Kebijaksanaan Palsu dari Para Filosof ”oleh Lacantius,“ Sebuah bola di mana orang-orang di sisi lain hidup dengan kaki mereka di atas kepala mereka, di mana hujan, salju dan hujan es jatuh ke atas, di mana pohon dan tanaman tumbuh terbalik dan langit lebih rendah dari tanah? Keajaiban kuno taman-taman gantung Babylon menyusut menjadi tidak ada apa-apanya dibandingkan dengan ladang, laut, kota dan pegunungan yang dipercaya oleh para filsuf kafir untuk bergelantungan dari bumi tanpa dukungan! ”

  • Ini adalah fallacy ‘appeal to emotion’. Memancing emosi pembaca untuk mempengaruhi pendapatnya.
  • Arah Atas dan Bawah

Gravitasi, Orbit, Pasang Laut (115-118)

115Hukum kepadatan dan daya apung yang ada dengan sempurna menjelaskan fisika benda-benda yang jatuh jauh sebelum Freemason ksatria“ Tuan ”Isaac Newton menganugerahkan teorinya tentang“ gravitasi ”ke dunia. Ini adalah fakta bahwa benda-benda yang ditempatkan dalam medium yang lebih padat naik sementara benda-benda yang ditempatkan di media yang kurang padat akan tenggelam. Agar sesuai dengan model heliosentris yang tidak naik atau turun, Newton malah mengklaim objek tertarik ke massa besar dan jatuh ke tengah. Tidak satu pun eksperimen dalam sejarah, bagaimanapun, telah menunjukkan sebuah objek yang cukup besar, berdasarkan massanya sendiri, menyebabkan massa yang lebih kecil lainnya tertarik padanya seperti Newton mengklaim “gravitasi” tidak dengan Bumi, Matahari, Bulan, Bintang dan Planet. “

116Belum pernah ada eksperimen tunggal dalam sejarah yang menunjukkan suatu objek yang cukup besar, berdasarkan massanya saja, menyebabkan massa yang lebih kecil untuk mengorbit di sekitarnya. Teori sihir gravitasi memungkinkan untuk lautan, bangunan dan orang-orang untuk selamanya menempel di bagian bawah bola berputar sementara secara bersamaan menyebabkan benda-benda seperti Bulan dan satelit untuk tetap terkunci di orbit melingkar abadi di sekitar Bumi. Jika keduanya benar maka orang harus dapat melompat dan mulai mengorbit lingkaran di sekitar Bumi, atau Bulan seharusnya sudah lama tersedot ke Bumi. Tidak satu pun dari teori-teori ini yang pernah diverifikasi secara eksperimental dan hasil dugaan mereka saling eksklusif. ”

117Newton juga berteori dan sekarang secara umum diajarkan bahwa pasang surut lautan Bumi disebabkan oleh daya tarik bulan gravitasi. Jika Bulan hanya berdiameter 2.160 mil dan Bumi 8.000 mil, bagaimanapun, menggunakan matematika dan “hukum” mereka sendiri, maka Bumi menjadi 87 kali lebih besar dan oleh karena itu objek yang lebih besar harus menarik yang lebih kecil untuk itu, dan tidak sebaliknya. Jika gravitasi bumi yang lebih besar adalah apa yang membuat Bulan berada di orbit, tidak mungkin gravitasi Bulan yang lebih kecil untuk menggantikan gravitasi Bumi, terutama di permukaan laut Bumi, di mana daya tarik gravitasinya akan semakin jauh melampaui Bulan. Dan jika gravitasi Bulan benar-benar menggantikan Bumi yang menyebabkan gelombang ditarik ke arahnya, seharusnya tidak ada yang menghentikan mereka dari terus maju dan ke atas menuju penarik besar mereka. ”

118Selanjutnya, kecepatan dan jalan Bulan adalah seragam dan karenanya harus memberikan pengaruh yang seragam pada pasang-surut Bumi, padahal sebenarnya pasang surut Bumi sangat bervariasi dan tidak mengikuti Bulan. Danau bumi, kolam, rawa-rawa, dan perairan pedalaman lainnya juga misterius tetap selamanya di luar pegangan gravitasi Bulan! Jika “gravitasi” benar-benar menggambar lautan Bumi hingga ke sana, semua danau, kolam dan badan air lainnya pasti juga memiliki pasang surut.

Matahari, Planet & Heliosentrisme (119-128)

119Diklaim bahwa planet-planet lain adalah bola dan oleh karena itu Bumi juga harus menjadi bola.Pertama, Bumi adalah “dataran” bukan “planet,” sehingga bentuk “planet” ini di langit tidak ada kaitannya dengan bentuk Bumi di bawah kaki kita. Kedua, “planet” ini telah dikenal selama ribuan tahun di seluruh dunia sebagai “bintang yang mengembara” karena mereka berbeda dari bintang tetap lainnya hanya dalam gerakan relatif mereka. Ketika melihat dengan mata telanjang tanpa prasangka atau melalui teleskop, bintang-bintang yang tetap dan berkelana muncul sebagai cakram cahaya bercahaya, BUKAN spora terra firma. Gambar-gambar dan video yang ditunjukkan oleh NASA dari planet-planet terra firma spora semuanya jelas merupakan gambar buatan komputer yang palsu, dan bukan foto. ”

120timologi dari kata“ planet ”sebenarnya berasal dari planete Inggris Kuno, dari planete Perancis Kuno (Planéte Perancis Modern), dari Planeta Latin, dari planet Yunani, dari (asteres) planetai“ mengembara (bintang), ”dari planasthai “Mengembara,” dari asal tidak diketahui, mungkin dari PIE * pele “datar, menyebar” atau gagasan “menyebar.” Dan Plane (n) “permukaan datar,” c. 1600, dari planum Latin “permukaan datar, bidang, tingkat, dataran,” planus “datar, datar, datar, polos, jelas.” Mereka hanya menambahkan “t” ke pesawat Bumi kita dan semua orang mempercayainya. ”

121Ketika Anda mengamati Matahari dan Bulan Anda melihat dua lingkaran berjarak sama yang sama yang melacak jalur serupa pada kecepatan yang sama di sekitar Bumi yang datar dan stasioner. “Para ahli” di NASA, bagaimanapun, mengklaim akal sehat Anda setiap hari adalah salah dalam segala hal! Untuk mulai dengan, mereka mengatakan Bumi tidak datar tetapi bola besar; tidak stasioner tetapi berputar sekitar 19 mil per detik; mereka mengatakan bahwa Matahari tidak berputar mengelilingi Bumi seperti yang terlihat, tetapi Bumi berputar mengelilingi Matahari; Bulan, di sisi lain, berputar mengelilingi Bumi, meskipun tidak Timur ke Barat seperti yang terlihat, agak Barat ke Timur; dan Matahari sebenarnya 400 kali lebih besar dari Bulan dan 400 kali lebih jauh! Anda dapat melihat dengan jelas bahwa mereka memiliki ukuran dan jarak yang sama, Anda dapat melihat Bumi datar, Anda dapat merasakan Bumi stasioner, tetapi menurut injil astronomi modern, Anda salah dan orang tolol yang layak dipermainkan jika Anda berani untuk mempercayai mata dan pengalaman Anda sendiri. ”

122Mengutip Allen Daves,” Jika Pemerintah atau NASA telah mengatakan kepada Anda bahwa Bumi stasioner, bayangkan itu.Dan kemudian bayangkan kita mencoba meyakinkan orang bahwa ‘tidak, tidak itu tidak diam, itu bergerak maju dengan kecepatan peluru 32 kali dan berputar pada 1.000 mil per jam.’Kami akan ditertawakan! Kami akan memiliki begitu banyak orang yang mengatakan kepada kami ‘kamu gila, Bumi tidak bergerak!’Kita akan diejek karena tidak memiliki dukungan ilmiah untuk teori Bumi bergerak yang berbelit-belit ini. Dan bukan hanya itu, tetapi kemudian orang-orang akan berkata, ‘oh, bagaimana Anda menjelaskan suasana tetap, tenang, dan gerakan Matahari yang bisa diamati, bagaimana Anda menjelaskannya?’ Bayangkan mengatakan kepada orang-orang, ‘tidak, tidak, atmosfer juga bergerak tetapi entah bagaimana secara ajaib direkatkan ke Bumi yang bergerak. Alasannya bukan hanya karena Bumi diam. ‘ Jadi apa yang sebenarnya kita lakukan adalah apa yang masuk akal.Kami mengatakan bahwa teori Bumi-bergerak itu tidak masuk akal. Teori Bumi stasioner masuk akal dan kita diejek. Anda harus membayangkan itu menjadi cara lain untuk menyadari betapa RIDICULOUS situasi ini. Teori ini dari Pemerintah dan NASA bahwa Bumi berputar dan mengorbit, serta bersandar dan bergoyang-goyang adalah omong kosong yang mutlak, namun orang-orang tetap mematuhinya, dengan erat, seperti boneka beruang.Mereka hanya tidak bisa membawa diri untuk menghadapi kemungkinan bahwa Bumi tidak bergerak meskipun SEMUA bukti menunjukkannya: kita tidak merasakan pergerakan, atmosfer belum terpesona, kita melihat Matahari bergerak dari Timur ke Barat, semuanya dapat dijelaskan oleh Bumi yang tidak bergerak tanpa memasukkan semua asumsi ini untuk menutupi asumsi sebelumnya menjadi buruk. “”

123Angka astronomi Heliocentrist selalu terdengar sangat tepat, tetapi mereka secara historis terkenal karena secara teratur dan secara drastis mengubah mereka agar sesuai dengan berbagai model mereka. Misalnya, pada waktunya Copernicus menghitung jarak Matahari dari Bumi menjadi 3,391,200 mil. Abad berikutnya Johannes Kepler memutuskan itu sebenarnya 12.376.800 mil jauhnya. Issac Newton pernah berkata, “Itu penting bukan apakah kita menganggapnya 28 atau 54 juta mil jauhnya karena akan melakukan hal yang sama!” Seberapa ilmiah !? Benjamin Martin menghitung antara 81 dan 82 juta mil, Thomas Dilworth mengklaim 93.726.900 mil, John Hind menyatakan positif 95.298.260 mil, Benjamin Gould mengatakan lebih dari 96 juta mil, dan Christian Mayer mengira itu lebih dari 104 juta! Flat-Earthers sepanjang zaman, sebaliknya, telah menggunakan sextants dan pesawat trigonometri untuk membuat perhitungan dan menemukan Matahari dan Bulan keduanya hanya sekitar 32 mil dengan diameter dan kurang dari beberapa ribu mil dari Bumi. ”

  • Teknologi berkembang, begitu juga akurasi dan ketepatan pengukuran tersebut.
  • Galat Pengukuran

124Rekaman balon amatir yang diambil di atas awan telah memberikan bukti visual yang menakjubkan bahwa Matahari tidak dapat jutaan mil jauhnya. Dalam beberapa bidikan Anda dapat melihat titik panas yang jelas yang merefleksikan awan tepat di bawah pengaruh sinar matahari. Jika Matahari benar-benar jutaan mil jauhnya, titik panas yang kecil dan terlokalisasi itu tidak dapat terjadi. ”

125Bukti lain bahwa Matahari tidak jutaan mil jauhnya ditemukan dengan menelusuri sudut sinar-sinar matahari kembali ke sumbernya di atas awan. Ada ribuan gambar yang menunjukkan bagaimana sinar matahari turun melalui awan-tertutup dengan berbagai sudut konvergen. Area konvergensi tentu saja Matahari, dan jelas BUKAN jutaan mil jauhnya, tetapi relatif relatif dekat dengan Bumi tepat di atas awan. ”

126Perjalanan tahunan Matahari dari daerah tropik ke tropik, titik balik matahari ke titik balik matahari, adalah apa yang menentukan panjang dan karakter hari, malam, dan musim. Inilah sebabnya mengapa daerah khatulistiwa mengalami hampir sepanjang musim panas dan panas sementara garis lintang yang lebih tinggi Utara dan terutama Selatan mengalami musim yang lebih berbeda dengan musim dingin yang keras. Model heliosentris mengklaim perubahan musim berdasarkan pada bola-Bumi yang diduga “kemiringan sumbu” dan “orbit elips” di sekitar Matahari, namun model saat ini menempatkan kita paling dekat dengan Matahari (91.400.000 mil) pada bulan Januari ketika musim dingin sebenarnya, dan terjauh dari Matahari (94.500.000 mil) pada bulan Juli ketika musim panas sebenarnya di sebagian besar Bumi. ”

127Fakta bahwa refleksi Matahari dan Bulan pada air selalu membentuk garis lurus dari cakrawala ke pengamat membuktikan Bumi bukanlah bola. Jika permukaan bumi melengkung, tidak mungkin cahaya yang dipantulkan melengkung di atas bola dari cakrawala menjadi pengamat. ”

128Ada jam matahari dan moissial batu berumur berabad-abad yang besar di seluruh dunia yang masih menunjukkan waktu sekarang sampai semenit seperti hari ketika mereka dibuat. Jika Bumi, Matahari, dan Bulan benar-benar tunduk pada jumlah gerakan berputar, berputar, bergoyang-goyang, dan berputar-putar yang diklaim oleh astronomi modern, tidak mungkin monumen-monumen ini secara akurat memberi tahu waktu tanpa penyesuaian konstan. ”

  • Waktu harian ditentukan oleh posisi Matahari di langit, bukan oleh faktor lain. Dengan demikian, jam Matahari akan selalu akurat. Perangkat pencatat waktu lain yang perlu dikalibrasi setiap waktu.

Gerak Bintang (129-130)

129“Mengutip William Carpenter,” Mengapa, atas nama akal sehat, harus pengamat harus memperbaiki teleskop mereka pada pangkalan batu yang kokoh agar mereka tidak menggerakkan keluasan rambut, – jika Bumi tempat mereka memperbaikinya bergerak di tingkat sembilan belas mil dalam sedetik? Memang, untuk percaya bahwa ‘enam ribu juta juta ton’ adalah ‘bergulir, melonjak, terbang, melesat melalui ruang untuk selama-lamanya’ dengan kecepatan dibandingkan dengan tembakan dari meriam adalah ‘pelatih yang sangat lambat,’ dengan tidak terputuskan seperti itu keakuratan bahwa teleskop yang dipasang pada pilar granit di observatorium tidak akan memungkinkan astronom bermata lynx untuk mendeteksi variasi dalam gerakan selanjutnya dari bagian seperseribu keliling rambut adalah untuk memahami keajaiban dibandingkan dengan semua keajaiban dalam catatan bersama-sama akan tenggelam menjadi tidak berarti. Karena kita dapat, (di garis lintang utara), lihat Bintang Utara, ketika melihat keluar dari jendela yang menghadapinya – dan keluar dari sudut yang sama dari panel kaca yang sama di jendela yang sama – sepanjang tahun , itu adalah bukti yang cukup bagi setiap orang dalam pikirannya bahwa kita tidak membuat gerakan sama sekali dan bahwa Bumi bukanlah bola dunia. ”

130“Dari“ Bumi Bukan Dunia! ”Oleh Samuel Rowbotham,“ Ambil dua tabung logam yang dijahit dengan hati-hati, tidak kurang dari enam kaki panjangnya, dan tempatkan mereka satu yard terbelah, di sisi berlawanan dari bingkai kayu, atau yang solid balok kayu atau batu: jadi sesuaikan mereka bahwa pusat atau poros penglihatan mereka harus sejajar satu sama lain. Sekarang, arahkan mereka ke bidang bintang tetap tertentu, beberapa detik sebelum waktu meridiannya. Biarkan pengamat ditempatkan di setiap tabung dan pada saat bintang muncul di tabung pertama, biarkan ketukan keras atau sinyal lainnya diberikan, untuk diulang oleh pengamat di tabung kedua ketika dia pertama kali melihat bintang yang sama. Jangka waktu yang berbeda akan berlalu antara sinyal yang diberikan. Sinyal akan mengikuti satu sama lain dalam suksesi yang sangat cepat, tetapi tetap, waktu antara cukup untuk menunjukkan bahwa bintang yang sama tidak terlihat pada saat yang sama oleh dua garis pandang paralel ketika hanya satu halaman terbelah. Kemiringan sedikit dari tabung kedua menuju tabung pertama akan diperlukan untuk bintang untuk dilihat melalui kedua tabung pada saat yang sama.Biarkan tabung tetap dalam posisi mereka selama enam bulan;pada akhir waktu, observasi atau eksperimen yang sama akan menghasilkan hasil yang sama – bintang akan terlihat pada waktu meridian yang sama, tanpa perubahan sedikit pun yang diperlukan ke arah tabung: dari mana dapat disimpulkan bahwa jika bumi telah pindah satu halaman tunggal di orbit melalui ruang angkasa, setidaknya akan diamati sedikit kemiringan tabung yang perbedaan dalam posisi satu yard sebelumnya diperlukan. Tetapi karena tidak ada perbedaan dalam arah tabung yang diperlukan, kesimpulannya tidak dapat dihindari, bahwa dalam enam bulan, sebuah meridian yang diberikan pada permukaan bumi tidak bergerak satu halaman, dan oleh karena itu, bahwa bumi tidak memiliki tingkat orbital sedikitpun. gerakan.”

Moon (131-135)

131“NASA dan astronomi modern mempertahankan bahwa Bulan adalah tempat tinggal yang padat, berbentuk bola, seperti Bumi yang benar-benar diterbangkan oleh manusia dan menginjakkan kaki. Mereka mengklaim Bulan adalah planetoid non-bercahaya yang menerima dan memantulkan semua cahayanya dari Matahari. Kenyataannya adalah, bagaimanapun, bahwa Bulan dapat diamati bukan tubuh yang solid, itu jelas melingkar, tetapi tidak bulat, dan tidak dengan cara apapun seperti planetoid Bumi yang manusia dapat menginjakkan kaki. Faktanya, Bulan telah terbukti sebagian besar transparan dan sepenuhnya bercahaya diri, bersinar dengan cahaya uniknya sendiri. ”

132“Cahaya Matahari berwarna keemasan, hangat, kering, pengawet dan antiseptik, sementara cahaya Bulan adalah perak, dingin, lembap, membusuk, dan septik. Sinar matahari menurunkan pembakaran api unggun, sementara sinar Bulan meningkatkan pembakaran. Zat-zat tumbuhan dan hewan yang terpapar sinar matahari cepat kering, mengecil, mengental, dan kehilangan kecenderungan untuk membusuk dan membusuk; anggur dan buah-buahan lainnya menjadi padat, sebagian manisan dan diawetkan seperti kismis, kurma, dan buah prem; daging hewan menggumpal, kehilangan konstituen gasnya yang mudah menguap, menjadi keras, kering, dan lambat membusuk. Ketika terkena sinar bulan, bagaimanapun, tanaman dan zat hewan cenderung menunjukkan gejala pembusukan dan pembusukan. Ini membuktikan bahwa cahaya Matahari dan Bulan berbeda, unik, dan berlawanan karena berada dalam model datar geosentris. ”

133“Di bawah sinar matahari langsung, termometer akan membaca lebih tinggi daripada termometer lain yang ditempatkan di tempat teduh, tetapi secara penuh, cahaya bulan langsung termometer akan membaca lebih rendah dari yang lain ditempatkan di tempat teduh. Jika cahaya Matahari dikumpulkan dalam lensa besar dan dilemparkan ke titik fokus, ia dapat menciptakan panas yang signifikan, sementara cahaya Bulan yang dikumpulkan sama tidak menciptakan panas. Dalam “Lancet Medical Journal,” dari 14 Maret 1856, khusus diberikan beberapa percobaan yang membuktikan sinar Bulan ketika terkonsentrasi benar-benar dapat mengurangi suhu pada termometer lebih dari delapan derajat. Jadi sinar matahari dan cahaya bulan jelas memiliki sifat yang sama sekali berbeda. ”

134“Selanjutnya Bulan itu sendiri tidak dapat secara fisik menjadi tubuh bulat dan reflektor cahaya Matahari. Reflektor harus rata atau cekung untuk sinar cahaya untuk memiliki sudut pandang apa pun; Jika permukaan reflektor cembung maka setiap sinar titik cahaya dalam garis langsung dengan radius tegak lurus ke permukaan sehingga tidak ada pantulan. ”

135“Bulan tidak hanya jelas bercahaya diri, menyinari cahaya uniknya sendiri, tetapi juga sebagian besar transparan. Ketika waxing atau bulan memudar terlihat pada siang hari adalah mungkin untuk melihat langit biru menembus Bulan. Dan pada malam yang cerah, selama siklus waxing atau memudarnya, bahkan dimungkinkan untuk sesekali melihat bintang dan “planet” secara langsung melalui permukaan Bulan!Royal Astronomical Society telah mencatat banyak kejadian seperti itu sepanjang sejarah yang semuanya menentang model heliosentris. ”

Gerhana (136-137)

136Banyak orang berpikir bahwa kemampuan astronomi modern untuk secara akurat memprediksi gerhana bulan dan matahari adalah hasil dan bukti positif dari teori heliosentris alam semesta. Namun faktanya adalah bahwa gerhana telah diprediksi secara akurat oleh budaya di seluruh dunia selama ribuan tahun sebelum “bola heliosentris-Bumi” bahkan merupakan secercah dalam imajinasi Copernicus.Ptolemeus pada abad ke-1 M secara akurat memprediksi gerhana selama enam ratus tahun atas dasar bumi yang datar dan stasioner dengan presisi yang sama dengan siapa pun yang hidup saat ini. Sepanjang perjalanan kembali pada 600 SM, Thales secara akurat memprediksi gerhana yang mengakhiri perang antara Medes dan Lidia. Gerhana terjadi secara teratur dengan presisi dalam siklus 18 tahun, jadi terlepas dari kosmologi geosentrik atau heliosentris, datar atau bola bumi, gerhana dapat dihitung secara akurat independen dari faktor-faktor tersebut. ”

137Asumsi lain dan bukti yang seharusnya dari bentuk Bumi, para heliocentris mengklaim bahwa gerhana bulan disebabkan oleh bayangan bola-Bumi yang menyamar Bulan. Mereka mengklaim bahwa Matahari, Bumi, dan Bulan sphere sempurna sejajar seperti tiga bola biliar berturut-turut sehingga cahaya Matahari melemparkan bayangan Bumi ke Bulan. Sayangnya untuk heliocentris, penjelasan ini benar-benar tidak valid karena fakta bahwa gerhana bulan telah terjadi dan terus terjadi secara teratur ketika Matahari dan Bulan masih terlihat bersama di atas cakrawala! Agar cahaya Matahari dapat menebarkan bayangan Bumi ke Bulan, tiga tubuh harus selaras dalam 180 derajat syzygy lurus, tetapi sedini masa Pliny, ada catatan gerhana bulan terjadi ketika Matahari dan Bulan terlihat di langit. Oleh karena itu eclipsor Bulan tidak bisa menjadi bayangan Bumi / Bumi dan beberapa penjelasan lain harus dicari. ”

Kenampakan Objek Berjarak Jauh (138-139)

138Bukti “favorit lain dari bola-Earthers adalah penampilan dari seorang pengamat di pantai lambung kapal yang dikaburkan oleh air dan menghilang dari pandangan ketika berlayar pergi menuju cakrawala. Klaim mereka adalah bahwa lambung kapal menghilang sebelum kepala tiang mereka karena kapal tersebut memulai deklinasinya di sekitar lengkungan bola Bumi. Sekali lagi, bagaimanapun, kesimpulan terburu-buru mereka diambil dari premis yang salah, yaitu bahwa hanya pada bola-Bumi bisa fenomena ini terjadi.Faktanya adalah bahwa Hukum Perspektif pada permukaan pesawat mendikte dan mengharuskan terjadinya hal yang sama persis. Misalnya seorang gadis yang mengenakan gaun berjalan menuju cakrawala akan tampak tenggelam ke Bumi semakin jauh dia berjalan. Kakinya akan menghilang dari pandangan pertama dan jarak antara tanah dan bagian bawah gaunnya akan berkurang secara bertahap sampai setelah sekitar setengah mil sepertinya bajunya menyentuh tanah saat dia berjalan di atas kaki yang tidak terlihat. Seperti halnya pada permukaan pesawat, bagian terendah dari objek yang surut dari suatu titik pengamatan harus menghilang sebelum yang tertinggi. ”

139Tidak hanya hilangnya lambung kapal yang dijelaskan oleh Hukum Perspektif pada permukaan datar, terbukti benar dengan bantuan teleskop yang baik. Jika Anda menonton kapal berlayar jauh ke cakrawala dengan mata telanjang sampai lunasnya benar-benar lenyap dari pandangan di bawah seharusnya “kelengkungan Bumi,” kemudian melihat melalui teleskop, Anda akan melihat seluruh kapal dengan cepat memperbesar kembali ke tampilan, lambung dan semua, membuktikan bahwa hilangnya itu disebabkan oleh Hukum Perspektif, bukan oleh dinding air yang melengkung! Ini juga membuktikan bahwa cakrawala hanyalah garis pandang menghilang dari sudut pandang Anda, BUKAN dugaan “kelengkungan” Bumi. “

Efek Coriolis (140-141)

140Pendulum Foucault sering dikutip sebagai bukti dari Bumi yang berputar tetapi setelah penyelidikan lebih dekat membuktikan sebaliknya. Untuk mulai dengan, pendulum Foucault tidak seragam berayun ke satu arah. Kadang-kadang mereka berputar searah jarum jam dan kadang-kadang berlawanan arah jarum jam, kadang-kadang mereka gagal untuk memutar dan kadang-kadang mereka memutar terlalu banyak.Perilaku pendulum sebenarnya bergantung pada 1) gaya awal yang memulai ayunannya dan, 2) sambungan ball-and-socket yang digunakan yang paling mudah memfasilitasi gerakan melingkar di atas yang lain. Rotasi Bumi yang seharusnya benar-benar tidak penting dan tidak relevan dengan ayunan pendulum.Jika dugaan rotasi konstan Bumi mempengaruhi pendulum dengan cara apa pun, maka tidak perlu secara manual memulai pendulum bergerak. Jika rotasi diurnal Bumi menyebabkan 360 derajat rotasi diurnal pendulum, maka seharusnya tidak ada pendulum stasioner di mana pun di Bumi! ”

141Efek Coriolis” sering dikatakan menyebabkan sink dan mangkuk toilet di Belahan Bumi Utara untuk mengalirkan berputar ke satu arah sementara di Belahan Bumi Selatan menyebabkan mereka berputar berlawanan arah, sehingga memberikan bukti bola yang berputar-Bumi. Namun, sekali lagi, seperti Pendulums Foucault yang berputar entah ke mana, bak cuci dan toilet di belahan Utara dan Selatan tidak berputar secara konsisten ke satu arah! Wastafel dan toilet di rumah yang sama sering ditemukan berputar arah berlawanan, tergantung sepenuhnya pada bentuk cekungan dan sudut masuknya air, bukan rotasi Bumi yang seharusnya. ”

  • Anggapan bahwa efek Coriolis menyebabkan air berputar di wastafel sebenarnya adalah mitos. Efek Coriolis tidak cukup kuat untuk menyebabkannya. Dubay menggunakan ini sebagai “bukti” merupakan fallacy ‘strawman’.

“Jika Bumi itu Datar” (142-143)

142Orang-orang mengklaim bahwa jika Bumi datar, mereka harus dapat menggunakan teleskop dan melihat jelas di lautan! Ini tidak masuk akal, karena udara penuh dengan curah hujan terutama di atas lautan, dan terutama pada lapisan atmosfer terendah dan paling padat TIDAK transparan.Bayangkan kabut buram di jalan pada hari yang panas dan lembab. Bahkan teleskop terbaik akan kabur jauh sebelum Anda bisa melihat melintasi lautan. Namun, Anda dapat menggunakan teleskop untuk memperbesar BANYAK Bumi kita yang lebih datar daripada yang mungkin dilakukan pada bola dengan keliling 25.000 mil. ”

  • Dia benar. Dan kita bisa melihat horizon yang sangat jelas jika jarak ke horizon berada dalam jangkauan jarak pandang kita dan cuaca sedang cerah. Jika Bumi datar, kita tidak akan pernah bisa melihat horizon yang jelas, dan horizon akan selalu terlihat kabur, walaupun cuaca cerah.
  • TBD(horizon-bumi-datar)

143Orang-orang mengklaim bahwa jika Bumi datar, dengan Matahari berputar-putar di sekeliling kita, kita harus dapat melihat Matahari dari mana-mana di seluruh Bumi, dan seharusnya ada siang hari bahkan di malam hari.Karena Matahari BUKAN 93 juta mil jauhnya tetapi hanya beberapa ribu dan bersinar seperti lampu sorot, begitu telah bergerak cukup jauh dari lokasi Anda menjadi tidak terlihat di luar cakrawala dan siang hari perlahan memudar sampai benar-benar menghilang. Jika Matahari berada sejauh 93 juta mil dan Bumi adalah bola yang berputar, transisi dari siang ke malam akan menjadi hampir seketika saat Anda melewati garis terminator. ”

Bulan (144-147)

144Gambar Bulan muncul terbalik di belahan bumi Selatan dan sisi kanan di Utara sering disebut sebagai bukti bola Bumi, tetapi sekali lagi, setelah diamati lebih dekat, berikan bukti lain dari model datar. Bahkan, fotografi selang waktu menunjukkan Bulan itu sendiri berputar searah jarum jam seperti roda saat berputar mengelilingi Bumi. Anda dapat menemukan gambar Bulan di 360 derajat berbagai kemiringan dari seluruh Bumi hanya tergantung di mana dan kapan foto itu diambil. ”

145Para Heliocentris percaya bahwa Bulan adalah sebuah bola, meskipun penampakannya jelas adalah sebuah piringan bercahaya datar. Kami hanya pernah melihat satu wajah yang sama (meskipun dengan berbagai kecenderungan) Bulan, namun diklaim bahwa ada “sisi gelap Bulan” yang masih tersembunyi. NASA menyatakan Bulan berputar berlawanan dengan putaran Bumi dengan cara tersinkronisasi sempurna sehingga gerakan-gerakan tersebut saling membatalkan satu sama lain sehingga kita tidak akan pernah bisa mengamati sisi gelap Bulan di luar gambar CGI palsu mereka yang mengerikan. Kenyataannya adalah, bagaimanapun, jika Bulan adalah bola, pengamat di Antartika akan melihat wajah yang berbeda dari mereka di khatulistiwa, namun mereka tidak – hanya wajah datar yang sama diputar pada berbagai derajat. ”

146Model bola-Bumi mengklaim Bulan mengorbit di sekitar Bumi setiap 28 hari sekali, namun jelas bagi siapa saja untuk melihat bahwa Bulan mengorbit mengelilingi Bumi setiap hari! Orbit Bulan sedikit lebih lambat daripada Matahari, tetapi mengikuti jalur Sun yang sama dari Tropic ke Tropic, titik balik matahari ke titik balik matahari, membuat lingkaran penuh di atas Bumi hanya kurang dari 25 jam.”

  • Jika kita berasumsi Bumi datar, maka hal itulah yang akan sekilas terlihat. Namun kita harus mengabaikan semua pengamatan lain yang bertentangan dengan asumsi tersebut, seperti fase bulan dan gerhana.

147Model bola-Bumi mengklaim bahwa Matahari adalah 400 kali lebih besar dari Bulan dan 400 kali lebih jauh dari Bumi membuat mereka” palsu “muncul persis dengan ukuran yang sama. Sekali lagi, model bola meminta kita untuk menerima sebagai sesuatu yang kebetulan yang tidak dapat dijelaskan selain oleh desain alami. Matahari dan Bulan menempati jumlah ruang yang sama di langit dan telah diukur dengan sekstan untuk memiliki ukuran yang sama dan jarak yang sama, jadi mengklaim sebaliknya adalah melawan mata, pengalaman, eksperimen, dan akal sehat kita. ”

Astronomi (148-151)

148Mengutip” Bumi Bukan Dunia! “Oleh Samuel Rowbotham,” Ditemukan oleh pengamatan bahwa bintang-bintang mencapai garis meridian sekitar empat menit sebelumnya setiap dua puluh empat jam daripada matahari, mengambil waktu matahari sebagai standar. Ini membuat 120 menit setiap tiga puluh hari, dan dua puluh empat jam dalam setahun. Oleh karena itu semua rasi bintang telah berlalu sebelum atau sebelum matahari pada waktu itu. Ini adalah fakta yang sederhana seperti yang diamati di alam, tetapi teori kekar dan gerak pada sumbu dan di orbit tidak memiliki tempat untuk itu. Kebenaran yang dapat dilihat harus diabaikan, karena teori ini menghalangi, dan mencegah para pemilihnya untuk memahaminya. ”

149Selama ribuan tahun rasi bintang yang sama tetap dalam pola yang sama tanpa keluar dari posisi apa pun. Jika Bumi adalah bola besar yang berputar di sekitar Matahari yang lebih besar berputar di sekitar galaksi besar yang memotret dari Biggest Bang seperti yang diklaim NASA, tidak mungkin rasi bintang itu akan tetap demikian. Berdasarkan model mereka, kita harus, pada kenyataannya, memiliki langit malam yang sangat berbeda setiap malam dan tidak pernah mengulangi pola bintang yang sama dua kali. ”

150Jika Bumi adalah bola yang berputar tidak mungkin untuk memotret jejak waktu bintang-bintang yang memutar lingkaran sempurna di sekitar Polaris di mana saja kecuali Kutub Utara. Pada semua sudut pandang lain bintang-bintang akan terlihat melakukan perjalanan lebih atau kurang horizontal di cakrawala pengamat karena gerakan 1000mph diduga di bawah kaki mereka. Kenyataannya, bagaimanapun, bintang-bintang di sekeliling Polaris dapat selalu difoto dengan memutar lingkaran sempurna di sekitar bintang pusat sampai ke Tropic of Capricorn. ”

151Jika Bumi adalah bola yang berputar mengelilingi Matahari maka sebenarnya tidak mungkin foto bintang-jejak untuk menunjukkan lingkaran yang sempurna bahkan di Kutub Utara! Karena Bumi juga diduga bergerak 67.000 mph di sekitar Matahari, Matahari bergerak 500.000 mph di sekitar Bima Sakti, dan seluruh galaksi akan 670.000.000php, empat gerakan kontradiktif ini akan membuat jejak waktu-bintang-bintang semua menunjukkan garis lengkung yang tidak beraturan. ”

Geodesi (152-153)

1522003, tiga profesor Universitas Geografi berkolaborasi dalam percobaan untuk membuktikan bahwa negara bagian Kansas memang benar-benar lebih datar daripada panekuk! Dengan menggunakan survei geodetik topigrafis yang mencakup lebih dari 80.000 mil persegi, ditetapkan bahwa Kansas memiliki rasio kerataan 0,9997 di seluruh negara bagian sementara pancake rata-rata, yang diukur dengan tepat menggunakan mikroskop laser confocal, masuk pada 0,957, membuat Kansas secara harfiah lebih datar daripada pancake. ”

  • Berdasarkan definisi, survei geodetik adalah “survei dari daerah yang luas dengan melakukan koreksi terhadap kelengkungan permukaan Bumi”
  • TBD(survei-geodetik)
  • TBD(kansas-pancake)

153Mengutip Pendeta“ Atlas Geografi Fisik ”Thomas Milner, kita menemukan bahwa,“ Daerah-daerah luas menunjukkan tingkat kematian yang sempurna, hampir tidak ada yang ada melalui 1.500 mil dari Carpathian ke Ural. Selatan Baltik negara ini sangat datar sehingga angin utara yang dominan akan mendorong perairan Stattiner Haf ke dalam mulut Oder, dan memberi sungai aliran mundur 30 atau 40 mil. Dataran Venezuela dan New Granada, di Amerika Selatan terutama di sebelah kiri Orinoco, disebut Ilanos, atau bidang datar. Seringkali dalam ruang 270 mil persegi permukaannya tidak bervariasi satu kaki pun. Amazon hanya jatuh 12 kaki dalam 700 mil terakhir dari jalurnya; La Plata hanya memiliki satu atau tiga puluh tiga inci satu mil. ”

Kelengkungan (154-156)

154 “Kamera luar menyelam Felix Baumgartner Red Bull menunjukkan jumlah“ kelengkungan Bumi ”yang sama dari permukaan ke tinggi lompat-tinggi yang membuktikannya sebagai lensa wide-angle mata ikan yang menipu, sementara kamera biasa di bagian dalam menunjukkan gambar yang sempurna. horizon datar, tingkat mata pada 128.000 kaki, yang hanya konsisten dengan bidang datar. “

155Beberapa orang mengklaim telah melihat kelengkungan Bumi di luar jendela pesawat mereka.Kaca yang digunakan di semua pesawat komersial, bagaimanapun, melengkung untuk tetap menyiram dengan badan pesawat. Hal ini menciptakan sedikit efek yang bercampur dengan bias konfirmasi kesalahan orang karena kelengkungan dugaan Bumi. Pada kenyataannya, fakta bahwa Anda dapat melihat cakrawala pada tingkat mata di 35.000 kaki di kedua jendela pelabuhan / kanan membuktikan Bumi datar. Jika Bumi adalah bola, tidak peduli seberapa besar, cakrawala akan tetap berada di tempatnya dan Anda harus melihat TURUN lebih jauh dan lebih jauh untuk melihat cakrawala sama sekali. Melihat langsung ke luar jendela pada ketinggian 35.000 kaki, Anda tidak akan melihat apa pun kecuali “luar angkasa” dari pelabuhan dan jendela kanan, karena Bumi / cakrawala seharusnya DI BAWAH Anda. Jika mereka terlihat pada tingkat mata di luar kedua jendela samping, itu karena Bumi itu datar! ”

156Orang-orang juga mengklaim melihat kelengkungan di Go Pro atau rekaman kamera ketinggian tinggi lainnya di cakrawala. Meskipun benar bahwa cakrawala sering muncul cembung dalam rekaman tersebut, itu sama sering muncul cekung atau datar tergantung pada kemiringan / gerakan kamera. Efeknya hanyalah distorsi karena lensa sudut lebar.Dalam koreksi lensa dan rekaman yang diambil tanpa teknologi sudut lebar, semua bidikan ketinggian langit amatir tampak sangat datar. ”

Atmosfer & Luar Angkasa (157-162)

157Jika” gravitasi “secara ajaib menyeret atmosfer bersama dengan bola Bumi yang berputar, itu berarti atmosfer di dekat khatulistiwa akan berputar di lebih dari 1000mph, atmosfer di atas garis lintang akan berputar sekitar 500mph, dan secara bertahap lebih lambat turun. ke kutub di mana atmosfer tidak akan terpengaruh pada 0mph. Namun kenyataannya, atmosfer di setiap titik di Bumi sama-sama tidak terpengaruh oleh kekuatan yang diduga ini, karena tidak pernah diukur atau dihitung dan terbukti tidak ada oleh kemampuan pesawat terbang untuk terbang tanpa arah ke mana pun tanpa mengalami perubahan atmosfer seperti itu. . “

158Jika” gravitasi “secara ajaib menyeret atmosfer bersama dengan bola Bumi yang berputar, itu berarti semakin tinggi ketinggian, semakin cepat suasana berputar harus berputar di sekitar pusat rotasi. Namun dalam kenyataan, jika ini terjadi maka hujan dan kembang api akan berperilaku sepenuhnya berbeda ketika mereka jatuh melalui atmosfer berputar yang semakin lambat dan lebih lambat. Balon udara panas juga akan dipaksa lebih cepat ke arah Timur saat mereka naik melalui kecepatan atmosfer yang semakin meningkat. ”

  • TBD(menembak-vertikal)

159Jika ada semakin cepat dan lebih cepat berputar suasana semakin tinggi ketinggian yang berarti itu harus tiba-tiba berakhir pada beberapa ketinggian utama di mana lapisan tercepat dari gravitasi berputar berpadu memenuhi non-berputar non-berputar non-atmosfer yang diharapkan tak terbatas ruang hampa udara! NASA tidak pernah menyebutkan apa ketinggian dari prestasi yang tidak mungkin ini yang diduga terjadi, tetapi dengan mudah secara filosofis disangkal oleh fakta sederhana bahwa vakuum tidak dapat terhubung dengan non-vacuums sembari mempertahankan sifat-sifat vakum – belum lagi, efek seperti transisi akan memiliki pada “kapal luar angkasa” roket akan menjadi bencana. “

160Tidak mungkin roket atau jenis mesin penggerak jet apa pun bekerja di ruang hampa udara tanpa atmosfer karena tanpa udara / atmosfer untuk mendorong tidak ada yang dapat mendorong kendaraan ke depan. Sebaliknya, roket dan pesawat ulang-alik akan dikirim berputar di sekitar porosnya sendiri tak terkendali ke segala arah seperti giroskop.Tidak mungkin untuk terbang ke Bulan atau pergi ke arah mana pun, terutama jika “gravitasi” itu nyata dan terus-menerus menyedot Anda menuju tubuh terpendek yang paling dekat. ”

161Jika Bumi benar-benar bola, tidak akan ada alasan untuk menggunakan roket untuk terbang ke” luar angkasa “karena hanya menerbangkan pesawat terbang lurus di ketinggian manapun cukup lama dan akan mengirim Anda ke luar angkasa.Untuk mencegah pesawat mereka terbang bersinggungan dengan bola-Bumi, pilot harus terus-menerus melakukan koreksi ke bawah, atau hanya dalam beberapa jam, pesawat komersial rata-rata yang menempuh 500mph akan hilang di “angkasa luar”. Fakta bahwa ini tidak pernah terjadi, cakrawala buatan tetap sejajar di ketinggian yang diinginkan pilot dan TIDAK memerlukan penyesuaian ke bawah konstan, membuktikan Bumi bukan bola. ”

162Semua NASA dan roket” badan luar angkasa “lainnya diluncurkan tidak pernah lurus ke depan.Setiap roket membentuk kurva parabola, memuncak, dan tak terelakkan mulai jatuh kembali ke Bumi. Roket yang dinyatakan “berhasil” adalah beberapa yang tidak meledak atau mulai jatuh terlalu cepat tetapi membuatnya keluar dari jangkauan pandangan penonton sebelum jatuh ke perairan yang terbatas dan pulih. Tidak ada ketinggian sihir di mana roket atau apa pun bisa naik, naik, naik dan kemudian tiba-tiba saja mulai “mengambang bebas” di angkasa. Ini semua ilusi fiksi ilmiah yang dibuat oleh kabel, layar hijau, kolam gelap, beberapa rambut yang dikeriting dan pesawat Zero-G. ”

NASA & Tuduhan Pemalsuan (163-165)

163NASA dan lembaga ruang angkasa lainnya telah ditangkap berkali-kali dengan gelembung udara yang membentuk dan melayang di rekaman” luar angkasa “resmi mereka. Astronot juga telah ditangkap menggunakan scuba-space-gear, menendang kaki mereka untuk bergerak, dan astronot Luca Parmitano bahkan hampir tenggelam ketika air mulai mengisi helmnya sementara diduga di “ruang-berjalan.” Diakui bahwa astronot melatih karena mereka “Ruang-berjalan” di fasilitas pelatihan bawah air seperti NASA “Neutral Buoyancy Lab,” tetapi apa yang jelas dari “gelembung ruang” mereka, dan kesalahan lainnya adalah bahwa semua “ruang-walk” rekaman resmi juga palsu dan difilmkan di bawah- air.”

164Analisis banyak video interior dari” Stasiun Luar Angkasa Internasional, “telah menunjukkan penggunaan trik kamera seperti layar hijau, baju zirah, dan bahkan rambut yang dikeriting liar untuk mencapai efek jenis gravitasi nol. Rekaman astronot yang tampaknya mengambang di gravitasi nol dari “stasiun ruang angkasa” mereka tidak dapat dibedakan dari cuplikan pesawat Zero-G “muntahan komet”.Dengan menerbangkan manuver parabolik, efek mengambang Zero-G ini dapat dicapai berulang kali kemudian diedit bersama.Untuk hasil tembakan yang lebih panjang, NASA telah ditangkap menggunakan kabel sederhana dan teknologi layar hijau. ”

165NASA mengklaim bahwa seseorang dapat mengamati Stasiun Luar Angkasa Internasional lewat di atas membuktikan keberadaannya, namun analisis” ISS “yang dilihat melalui kamera zoom membuktikannya sebagai beberapa jenis hologram / drone, bukan dasar ruang mengambang fisik.Seperti yang Anda lihat dalam film dokumenter saya “ISS Hoax,” ketika memperbesar / memperkecil, “ISS” secara dramatis dan tidak mungkin mengubah bentuk dan warna, menampilkan efek pelangi prismatik hingga menjadi fokus seperti menyalakan atau mematikan TV lama. “

Satelit (166-171)

166The” satelit komunikasi geostasioner “pertama kali diciptakan oleh penulis fiksi ilmiah Freemason Arthur C. Clarke dan konon menjadi fakta sains hanya satu dekade kemudian. Sebelum ini, radio, televisi, dan sistem navigasi seperti LORAN dan DECCA sudah mapan dan berfungsi baik hanya dengan menggunakan teknologi berbasis darat. Saat ini kabel serat-optik yang besar menghubungkan internet melintasi lautan, menara sel raksasa yang melakukan pelacakan sinyal GPS, dan propagasi ionosfer memungkinkan gelombang radio terpental semua tanpa bantuan buku laris fiksi sains yang dikenal sebagai “satelit.” ”

167Satelit diduga mengapung di dalam termosfer di mana suhu diklaim naik dari 4,530 derajat Fahrenheit. Logam yang digunakan dalam satelit, bagaimanapun, seperti aluminium, emas dan titanium memiliki titik leleh 1,221, 1,948, dan 3,034 derajat masing-masing, semua jauh lebih rendah daripada yang mungkin mereka tangani. ”

168“Telepon“ satelit ”telah ditemukan memiliki masalah penerimaan di negara-negara seperti Kazakhstan dengan sangat sedikit menara telepon seluler. Jika Bumi adalah bola dengan 20.000 lebih satelit di sekitarnya, pemadaman seperti ini seharusnya tidak terjadi secara teratur di daerah pedesaan manapun. ”

  • Ini pernyataan yang tidak berdasar. Telepon satelit telah terbukti dapat digunakan di lokasi tanpa BTS telepon seluler, seperti di tengah lautan.

169“Piring TV“ satelit ”hampir selalu diposisikan pada sudut 45 derajat menuju menara repeater berbasis darat terdekat. Jika antena TV benar-benar mengambil sinyal dari satelit seluas lebih dari 100 mil di angkasa, sebagian besar antena TV akan mengarah lebih atau kurang lurus ke langit. Fakta bahwa piring “satelit” tidak pernah menunjuk lurus ke atas dan hampir selalu diposisikan pada sudut 45 derajat membuktikan mereka mengambil sinyal menara berbasis darat dan bukan “satelit luar angkasa”. ”

  • Satelit geostasioner selalu berada di atas khatulistiwa. Sebuah antena penerima di utara khatulistiwa harus diarahkan ke selatan, dan juga sebaliknya.

170Orang-orang bahkan mengklaim melihat satelit dengan mata telanjang mereka, tetapi ini konyol mengingat mereka lebih kecil dari bus dan diduga 100+ mil jauhnya; Tidak mungkin untuk melihat sesuatu yang sangat kecil sejauh itu. Bahkan dengan menggunakan teleskop, tidak ada yang mengklaim untuk melihat bentuk satelit melainkan menggambarkan melihat lewatnya lampu yang bergerak, yang bisa dengan mudah menjadi beberapa hal dari pesawat ke drone hingga bintang jatuh atau benda terbang tak dikenal lainnya. ”

  • Gerak satelit dapat diprediksi dan kita dapat mengetahui jauh hari sebelumnya kapan dan di mana satelit akan terlihat. Prediksi semacam itu hanya dapat dibuat menggunakan model Bumi bulat.

171NASA mengklaim ada lebih dari 20.000 satelit yang mengambang di sekitar atmosfer atas Bumi mengirimi kami radio, televisi, GPS, dan mengambil gambar dari planet ini. Semua gambar satelit ini, bagaimanapun, memang diakui “gambar komposit, diedit di photoshop!” Mereka mengklaim untuk menerima “pita pencitraan” dari satelit yang kemudian harus disambung bersama untuk membuat gambar komposit Bumi, yang semuanya jelas CGI dan bukan foto. Jika Bumi benar-benar bola dengan 20.000 satelit yang mengorbit, itu akan menjadi masalah sederhana untuk memasang kamera dan mengambil beberapa foto nyata. Fakta bahwa tidak ada foto satelit nyata dari bola bumi yang diduga ada untuk mendukung “pita citra CG komposit NASA”, adalah bukti lebih lanjut bahwa kita tidak diberitahu kebenaran. ”

Citra Bumi (172-178)

172 “Jika kamu memilih awan di langit dan mengawasi selama beberapa menit, dua hal akan terjadi: awan akan bergerak dan mereka akan berubah bentuk secara bertahap.Dalam rekaman resmi NASA tentang bola berputar Bumi, seperti video “selang waktu” Galileo, awan terus ditampilkan selama 24 jam setiap kalinya dan tidak bergerak atau berubah bentuk apa pun!Ini benar-benar mustahil, bukti lebih lanjut bahwa NASA menghasilkan video CGI palsu, dan bukti lebih lanjut bahwa Bumi bukanlah bola yang berputar. ”

  • Ini salah. Kita bisa melihat pergerakan awan jika kita perhatikan dengan teliti. Lebih sulit untuk melihatnya karena rekaman Galileo dalam resolusi rendah. Dengan rekaman resolusi yang lebih tinggi, seperti dari Himawari-8, kita dapat dengan mudah melihat gerakan awan.

173NASA memiliki beberapa foto dugaan bola-Bumi yang menunjukkan beberapa pola awan duplikat yang tepat! Kemungkinan memiliki dua atau tiga awan dengan bentuk yang sama dalam gambar yang sama kemungkinannya menemukan dua atau tiga orang dengan sidik jari yang sama persis. Sebenarnya ini adalah bukti kuat bahwa awan-awan itu disalin dan ditempelkan dalam program komputer dan bahwa gambar-gambar yang menunjukkan Bumi berbentuk bola adalah palsu. ”

174Seniman grafis NASA telah menempatkan hal-hal seperti wajah, naga, dan bahkan kata“ SEX ”ke dalam pola awan di atas berbagai bola-gambar Bumi mereka. Foto-foto Pluto 2015 terbaru mereka bahkan jelas memiliki gambar Disney “Pluto” anjing berlapis ke latar belakang. Kecurangan terang-terangan seperti itu tidak disadari oleh massa terhipnotis, tetapi memberikan bukti lebih lanjut tentang tidak sahnya NASA dan mitos planet berputar bola mereka. ”

175Para analis foto profesional telah membedah beberapa gambar NASA tentang bola Bumi dan menemukan bukti penyuntingan komputer yang tak terbantahkan. Misalnya, gambar Bumi yang diduga diambil dari Bulan telah terbukti disalin dan ditempel di, sebagaimana dibuktikan oleh potongan persegi panjang yang ditemukan di latar belakang hitam di sekitar “Bumi” dengan menyesuaikan tingkat kecerahan dan kontras. Jika mereka benar-benar di Bulan dan Bumi benar-benar bola, tidak perlu memalsukan gambar seperti itu. ”

  • Pepatah Hyman
  • Kompresi JPEG
  • Ada kemungkinan gambar tersebut bukan gambar asli dari NASA, dan kaum Bumi datar sendirilah yang melakukan modifikasi gambar. Kasus semacam ini sering terjadi di komunitas korban Bumi datar.

176 “Ketika citra NASA tentang bola-Bumi dibandingkan satu sama lain, warna tanah / lautan dan ukuran relatif benua secara konsisten sangat berbeda secara drastis satu sama lain untuk membuktikan tanpa keraguan bahwa semua foto itu palsu. ”

177“Dalam film dokumenter” A Funny Thing Happened on the Way to the Moon, “Anda dapat menyaksikan rekaman resmi NASA yang bocor yang menunjukkan Apollo 11 astronot Buzz Aldrin, Neil Armstrong dan Michael Collins, selama hampir satu jam, menggunakan transparansi dan trik kamera untuk tembakan palsu dari Bumi yang bulat! Mereka berkomunikasi melalui audio dengan kontrol di Houston tentang cara akurat memotret bidikan, dan seseorang terus mendorong mereka tentang cara memanipulasi kamera secara efektif untuk mencapai efek yang diinginkan. Pertama, mereka menghitamkan semua jendela kecuali yang menghadap ke bawah, yang mengarahkan kamera ke arah beberapa kaki. Ini menciptakan ilusi Bumi berbentuk bola yang dikelilingi oleh kegelapan ruang, padahal sebenarnya itu hanyalah jendela bundar di kabin gelap mereka. Neil Armstrong mengklaim pada titik ini adalah 130.000 mil dari Bumi, setengah jalan ke Bulan, tetapi ketika trik kamera selesai, penonton dapat melihat sendiri bahwa astro-nots tidak lebih dari beberapa lusin mil di atas permukaan Bumi, kemungkinan terbang di dataran tinggi!

178Orang-orang mengklaim Google Earth, entah bagaimana, membuktikan model bola tanpa menyadari bahwa Google Earth hanyalah sebuah program gabungan dari gambar yang diambil dari pesawat ketinggian tinggi dan kamera mobil jalanan yang ditumpangkan ke model CGI dari bola bumi. Hal yang sama bisa dengan mudah dimodelkan ke Bumi persegi atau bentuk lain dan oleh karena itu tidak dapat digunakan sebagai bukti kebusukan Bumi. ”

  • Ini adalah fallacy ‘strawman’. Google Earth bukanlah bukti Bumi berbentuk bulat.
  • Walaupun demikian, Google Earth dapat kita gunakan untuk memastikan jarak dan sebagainya, dan dengan informasi yang cukup, kita dapat memastikan bahwa Google Earth dapat mewakili Bumi yang sesungguhnya.

Durasi Penerbangan (179-184)

179Jika Bumi terus berputar ke arah timur 1000mph maka jangka waktu penerbangan pesawat yang menuju ke arah Timur vs. Barat harus berbeda secara signifikan.Jika pesawat komersial rata-rata menempuh 500 mph, maka penerbangan khatulistiwa ke arah Barat harus mencapai tujuan mereka kira-kira tiga kali kecepatan sebagai penerbangan kembali ke Timur. Namun kenyataannya, perbedaan dalam jangka waktu penerbangan Timur / Barat biasanya hanya dalam hitungan menit, dan tidak ada apa pun yang mendekati apa yang akan terjadi pada bola bumi yang berputar 1000mph. ”

180Model bola berputar menyatakan bahwa Bumi dan atmosfer akan bergerak bersama-sama di sekitar 500mph di mid-garis lintang di mana penerbangan LA ke NYC berlangsung. Pesawat komersial rata-rata yang bepergian 500mph membutuhkan waktu 5,5 jam perjalanan ke Timur dengan dugaan rotasi Bumi, sehingga penerbangan kembali Barat hanya perlu 2,75 jam, tetapi pada kenyataannya kita menemukan NYC rata-rata untuk penerbangan LA membutuhkan waktu 6 jam, waktu penerbangan benar-benar tidak konsisten dengan model bola berputar. “

181 “Penerbangan ke arah timur dengan dugaan putaran bola Bumi dari Tokyo ke LA membutuhkan rata-rata 10,5 jam, oleh karena itu penerbangan kembali ke arah barat terhadap dugaan spin harus mengambil rata-rata 5,25 jam, tetapi dalam kenyataannya mengambil rata-rata 11,5 jam, waktu penerbangan lain benar-benar tidak konsisten dengan model bola berputar. “

182 “Penerbangan ke arah timur dengan dugaan putaran bola Bumi dari NY ke London memakan waktu rata-rata 7 jam, oleh karena itu penerbangan balik ke Barat melawan dugaan putaran harus mengambil rata-rata 3,5 jam, tetapi dalam kenyataannya mengambil rata-rata 7,5 jam, waktu penerbangan sama sekali tidak konsisten dengan model bola berputar. ”

183 “Penerbangan ke arah timur dari Chicago ke Boston dengan dugaan putaran bola Bumi membutuhkan rata-rata 2,25 jam, oleh karena itu penerbangan balik ke Barat melawan dugaan putaran harus mengambil rata-rata lebih dari satu jam, tetapi dalam kenyataannya mengambil rata-rata 2,75 jam, sekali lagi, sama sekali tidak konsisten dengan model bola berputar. ”

184 “Penerbangan ke timur dari Paris ke Roma dengan dugaan putaran bola Bumi membutuhkan rata-rata 2 jam, oleh karena itu penerbangan balik ke Barat melawan dugaan putaran harus mengambil rata-rata 1 jam, tetapi dalam kenyataannya memiliki durasi penerbangan rata-rata 2 jam 10 menit, waktu penerbangan sama sekali tidak konsisten dengan model bola berputar. ”

Gerak & Bentuk Bumi (185-188)

185Kita diberitahu bahwa Bumi dan atmosfer berputar bersama pada kecepatan seragam yang sempurna yang tidak pernah dilihat, dirasakan, dirasakan, atau diukur oleh seseorang yang pernah merasakan gerakan 1000mph. Ini kemudian sering dibandingkan dengan bepergian dalam mobil dengan kecepatan seragam, di mana kita hanya merasakan gerakan saat akselerasi atau deselerasi. Namun dalam kenyataan, bahkan dengan mata tertutup, jendela naik, di atas tar halus di mobil mewah dengan hanya 50mph seragam, gerakan itu benar-benar bisa dirasakan!Pada 20 kali kecepatan ini, putaran imajiner 1000 mph Bumi pasti akan terlihat, dirasakan, dilihat dan didengar oleh semua orang. ”

  • Gerak mobil tidak memiliki kecepatan yang konstan. Jalan tidak mulus sempurna dan mesinnya bergetar, walaupun mobil melaju dengan kecepatan konstan. Jika kita dapat merasakan, maka kita dipengaruhi oleh percepatan (perubahan kecepatan).

186Orang-orang yang sensitif terhadap penyakit gerakan merasakan ketidaknyamanan yang nyata dan ketidaknyamanan fisik dari gerakan yang sesedikit lift atau naik kereta. Ini berarti bahwa dugaan serpihan Bumi yang diperkirakan sebesar 1000 mph tidak berpengaruh pada orang-orang seperti itu, tetapi tambahkan kecepatan seragam 50mph ekstra dari sebuah mobil dan perut mereka mulai memutar simpul. Gagasan bahwa penyakit gerakan tidak terlihat pada siapa pun pada 1000mph, tetapi tiba-tiba muncul pada 1050mph adalah konyol dan membuktikan Bumi tidak bergerak sama sekali. ”

  • Mabuk perjalanan terjadi karena ketidaksesuaian antara gerak yang dilihat dan yang dirasakan oleh sistem vestibular. Sistem vestibular mendeteksi percepatan, bukan kecepatan. Bumi tak memilki percepatan selain percepatan gravitasi dan percepatan sentrifugal.
  • Kemampuan Kita Untuk Merasakan Percepatan dan Kecepatan

187Hukum kedua termodinamika, atau dikenal sebagai hukum entropi, bersama dengan prinsip-prinsip dasar gesekan / perlawanan menentukan ketidakmungkinan Bumi menjadi bola berputar seragam. Seiring waktu, bola berputar Bumi akan mengalami jumlah drag yang dapat diukur secara konstan memperlambat putaran dan memperpanjang jumlah jam per hari. Seperti tidak sedikit perubahan seperti yang pernah diamati dalam semua sejarah yang tercatat itu tidak masuk akal untuk mengasumsikan Bumi pernah bergerak satu inci.”

  • Ruang angkasa itu hampa dan praktis tak memiliki materi. Sehingga tidak menimbulkan gesekan yang menghambat rotasi Bumi.
  • Rotasi bumi sebenarnya melambat karena adanya interaksi pasang dengan Bulan. Akibat interaksi pasang, energi kinetik rotasi Bumi  ditransfer ke Bulan, menyebabkan Bulan bergerak lebih cepat dan perlahan-lahan menjauh dari kita.

188Selama bertahun-tahun NASA telah dua kali mengubah kisah mereka tentang bentuk Bumi. Pada awalnya mereka mempertahankan Bumi adalah bola yang sempurna, yang kemudian berubah menjadi “oblate spheroid” diratakan di kutub, dan kemudian berubah lagi menjadi “berbentuk buah pir” sebagai belahan Selatan diduga membengkak keluar juga. Sayangnya untuk NASA, bagaimanapun, tidak ada foto resmi mereka yang menunjukkan oblate spheroid atau bumi berbentuk buah pir! Semua foto mereka, bertentangan dengan kata-kata mereka, menunjukkan Bumi yang bulat (dan jelas CGI palsu)”

Kitab Suci, Pengetahuan Kuno & Teori Konspirasi (189-194)

189“Alkitab, Alquran, Srimad Bhagavatam, dan banyak buku suci lainnya menggambarkan dan menunjukkan keberadaan Bumi yang datar dan datar yang geosentris. Misalnya, 1 Tawarikh 16:30 dan Mazmur 96:10 keduanya membaca, “Dia telah memperbaiki firma bumi, tidak dapat bergerak.” Dan Mazmur 93: 1 mengatakan, “Dunia juga dibangun, bahwa itu tidak dapat dipindahkan.” Alkitab juga berulang kali menegaskan bahwa Bumi “terentang” sebagai sebuah pesawat, dengan langit yang terentang di mana-mana di atas (tidak semuanya) memberikan bukti alkitabiah bahwa Bumi bukanlah bola yang berputar. ”

  • Bentuk dan gerak Bumi adalah hal yang dapat kita tentukan berdasarkan observasi pada Bumi, dan bukan bagian dari keyakinan.
  • Beberapa agama bergantung pada pengetahuan tentang bentuk Bumi, yang bulat, untuk melakukan ritual mereka.
  • Arah Kiblat di Bumi Bulat

190Budaya di seluruh dunia sepanjang sejarah telah menggambarkan dan mengakui keberadaan Bumi datar geosentris, stasioner. Mesir, India, Maya, Cina, penduduk asli Amerika dan secara harfiah setiap peradaban kuno di Bumi memiliki kosmologi datar-Bumi geosentris. Sebelum Pythagoras, gagasan tentang bola berputar-Bumi tidak ada dan bahkan setelah Pythagoras tetap menjadi pandangan minoritas yang tidak jelas sampai 2000 tahun kemudian ketika Copernicus mulai menghidupkan kembali teori heliosentris. ”

  • Hal tersebut membuktikan bahwa peradaban manusia telah berkembang. Namun kaum Bumi datar gagal untuk mengikuti perkembangan tersebut.

191Dari Pythagoras ke Copernicus, Galileo dan Newton, ke astronot modern seperti Aldrin, Armstrong dan Collins, kepada direktur NASA dan Grand Commander dari tingkat ke-33 C. Fred Kleinknecht, bapak pendiri dari mitos bola berputar semuanya adalah Freemason! Fakta bahwa begitu banyak anggota ini, masyarakat rahasia terbesar dan tertua yang ada semuanya telah menjadi konspirator yang mewujudkan “revolusi planet” literal ini di luar kemungkinan kebetulan dan memberikan bukti kolusi yang terorganisasikan dalam menciptakan dan memelihara multi-sektor ini. penipuan generasi. ”

192“Mengutip“ Terra Firma ”oleh David Wardlaw Scott,“ Sistem alam semesta, seperti yang diajarkan oleh para astronom modern, yang sepenuhnya didasarkan pada teori, untuk kebenaran di mana mereka tidak dapat memajukan satu pun bukti nyata, mereka telah memusnahkan diri mereka sendiri. dalam konspirasi diam, dan menolak untuk menjawab keberatan apa pun yang dapat dilakukan terhadap hipotesis mereka … Copernicus sendiri, yang menghidupkan kembali teori filsuf kafir Pythagoras, dan eksponen besarnya Sir Isaac Newton, mengakui bahwa sistem mereka dari Bumi yang berputar adalah hanya kemungkinan, dan tidak dapat dibuktikan oleh fakta. Hanya para pengikut mereka yang menghiasinya dengan nama ‘ilmu pasti’, ya, menurut mereka, ‘yang paling tepat dari semua ilmu.’ Namun, salah satu Astronom Royal untuk Inggris pernah berkata, berbicara tentang gerakan seluruh sistem Tata Surya: ‘Masalahnya dibiarkan dalam keadaan ketidakpastian yang paling menyenangkan, dan saya akan sangat senang jika ada yang dapat membantu saya keluar dari sana.’ Apa posisi yang sangat menyedihkan untuk ‘ilmu pasti’ yang ada di dalamnya adalah ini! ”

  • Salah apabila dikatakan tak satupun bukti nyata. Ini adalah klaim salah yang lingkupnya sangat luas, dan tak dapat dibantah secara spesifik. Bukti ada sangat banyak, pembaca dipersilakan untuk membaca bagian lain dari situs web kami ini.

193“Tidak ada anak atau laki-laki yang tidak diindoktrinasi dalam pikiran kanan mereka akan pernah menyimpulkan atau bahkan hamil diberikan kepada perangkat mereka sendiri, berdasarkan pengamatan pribadi mereka sendiri, bahwa Bumi adalah bola berputar berputar mengelilingi Matahari! Teori-teori imajinatif yang tidak ada di mana pun dalam pengalaman sehari-hari membutuhkan dan telah membutuhkan sejumlah besar propaganda konstan untuk menegakkan ilusi. “

  • Ada banyak observasi dan eksperimen yang bisa kita lakukan sendiri untuk menentukan bentuk Bumi.Beberapa di antaranya tercantum di sini: Observasi & Eksperimen Mudah
  • Tak dibutuhkan indoktrinasi dan propaganda untuk menyebarkan informasi yang sesuai dengan fakta, kecuali jika kita ingin menanamkan informasi yang salah, seperti konsep Bumi datar.

194“Dari David Wardlaw Scott,” Saya ingat diajar ketika seorang bocah, bahwa Bumi adalah bola yang hebat, berputar dengan sangat cepat mengelilingi Matahari, dan, ketika saya menyatakan kepada guru saya, saya takut bahwa air lautan akan jatuh, saya diberitahu bahwa mereka dicegah dari melakukannya oleh hukum Gravitasi Newton yang besar, yang membuat semuanya tetap di tempatnya. Saya menganggap bahwa wajah saya pasti menunjukkan tanda-tanda ketidakpercayaan, karena guru saya segera menambahkan – saya dapat menunjukkan kepada Anda bukti langsung dari ini; seorang lelaki bisa berputar di sekitar kepalanya ember berisi air tanpa tumpahannya, dan dengan cara yang sama, lautan bisa dibawa mengelilingi Matahari tanpa kehilangan setetes pun. Karena ilustrasi ini jelas dimaksudkan untuk menyelesaikan masalah ini, saya kemudian mengatakan tidak lagi pada subjek. Seandainya seperti itu diajukan kepada saya setelah itu sebagai seorang laki-laki, saya akan menjawab agak sebagai berikut – Pak, saya mohon untuk mengatakan bahwa ilustrasi yang Anda berikan tentang seorang pria mengitari ember air di sekeliling kepalanya, dan lautan berputar mengelilingi Matahari , tidak dalam derajat apa pun mengkonfirmasi argumen Anda, karena air dalam dua kasus ditempatkan di bawah keadaan yang sepenuhnya berbeda, tetapi, untuk menjadi nilai apa pun, kondisi dalam setiap kasus harus sama, yang di sini tidak. Ember adalah bejana hampa yang menahan air di dalamnya, sedangkan, menurut ajaran Anda, Bumi adalah bola, dengan kelengkungan terus-menerus di luar, yang, sesuai dengan hukum alam, tidak dapat menahan air apa pun. “”

  • Gurunya memang salah menggunakan analogi semacam itu. Namun hal tersebut bukanlah “bukti” Bumi datar, dan tak membutuhkan bantahan.

Percepatan (195-196)

195“Astronom mengatakan magnet daya tarik magis adalah apa yang membuat semua samudra dunia menempel pada bola-Bumi. Mereka mengklaim bahwa karena Bumi begitu masif, berdasarkan massa ini ia menciptakan kekuatan sihir yang mampu menahan orang, samudera dan atmosfer yang menempel erat di bagian bawah bola yang berputar. Sayangnya, bagaimanapun, mereka tidak dapat memberikan contoh praktis ini pada skala yang lebih kecil dari planet. Bola tenis basah yang berputar, misalnya, memiliki efek berlawanan dari bola-Bumi yang seharusnya! Air yang dituangkan ke atasnya hanya jatuh dari sisi, dan memberikan hasil berputar dalam air yang terbang 360 derajat seperti anjing yang berguncang setelah mandi. Para astronom mengakui bahwa contoh bola tenis basah menunjukkan efek sebaliknya dari bola Bumi mereka, tetapi mengklaim bahwa pada beberapa massa yang tidak diketahui, sifat perekat sihir gravitasi tiba-tiba menendang memungkinkan bola tenis basah yang membasahi Bumi untuk menjaga setiap tetes “gravitasi “Air menempel ke permukaan. Ketika teori yang belum terbukti ini bertentangan dengan semua eksperimen, pengalaman dan akal sehat, sudah saatnya untuk menjatuhkan teori. ”

196“Mengutip Marshall Hall,” Singkatnya, matahari, bulan, dan bintang-bintang benar-benar melakukan apa yang semua orang di seluruh sejarah telah melihatnya. Kami tidak percaya apa yang dikatakan mata kami karena kami telah diajarkan sistem palsu yang menuntut bahwa kami percaya apa yang belum dikonfirmasi dengan observasi atau percobaan. Sistem palsu itu menuntut agar Bumi berputar pada ‘sumbu’ setiap 24 jam dengan kecepatan lebih dari 1000 MPH di khatulistiwa. Tidak ada yang pernah, pernah, pernah melihat atau merasakan gerakan seperti itu (tidak melihat atau merasakan kecepatan 67.000MPH dari dugaan orbit Bumi di sekitar matahari atau 500.000 MPH dugaan kecepatannya di sekitar galaksi atau mundurnya dari dugaan ‘Big Bang’ di lebih dari 670.000.000 MPH!). Ingat, tidak ada eksperimen yang menunjukkan bumi bergerak. Tambahkan fakta bahwa dugaan kecepatan rotasi yang kita semua telah diajarkan sebagai fakta ilmiah HARUS menurun setiap inci atau mil satu pergi ke utara atau selatan khatulistiwa, dan itu menjadi jelas bahwa hal-hal seperti pemboman udara yang akurat dalam Perang Dunia II (turun cerobong asap dari 25.000 kaki dengan pesawat terbang ke arah mana pun dengan kecepatan tinggi) tidak akan mungkin jika dihitung di bumi bergerak di bawah pada beberapa ratus MPH dan berubah secara konstan dengan garis lintang. ”

Filsafat dan Teori Konspirasi (197-200)

197“Beberapa orang menyatakan bahwa tidak ada motif untuk penipuan skala besar dan datar atau bola itu tidak ada bedanya. Dengan memindahkan Bumi dari pusat alam semesta yang tak bergerak, para Mason ini telah memindahkan kita secara fisik dan metafisik dari suatu tempat yang sangat penting ke salah satu ketidakpedulian nihilistik yang lengkap. Jika Bumi adalah pusat Alam Semesta, maka ide-ide tentang Tuhan, ciptaan, dan tujuan untuk eksistensi manusia adalah gemilang. Tetapi jika Bumi hanyalah salah satu dari miliaran planet yang berputar di sekitar milyaran bintang dalam miliaran galaksi, maka ide-ide tentang Tuhan, ciptaan, dan tujuan khusus untuk Bumi dan keberadaan manusia menjadi sangat tidak masuk akal. Dengan diam-diam mengindoktrinasi kita ke dalam pemuja materialis ilmiah Sun mereka, bukan hanya kita kehilangan kepercayaan pada apa pun di luar materi, kita mendapatkan keyakinan mutlak dalam materialitas, kedangkalan, status, keegoisan, hedonisme, dan konsumerisme. Jika tidak ada Tuhan, dan semua orang hanya kecelakaan, maka yang terpenting adalah saya, saya, saya. Mereka telah mengubah Madonna, Bunda Allah, menjadi seorang gadis material yang hidup di dunia material. Perusahaan-perusahaan mereka yang kaya dan kuat dengan logo-logo Sun-cult yang apik menjual kita berhala untuk disembah, secara perlahan mengambil alih dunia sementara kita diam-diam mempercayai “ilmu pengetahuan” mereka, memilih politisi mereka, membeli produk mereka, mendengarkan musik mereka, dan menonton film mereka, mengorbankan jiwa kita di altar materialisme. Mengutip Morris Kline, “Teori heliosentris, dengan meletakkan matahari di pusat alam semesta… membuat manusia tampak sebagai salah satu dari kemungkinan pengembara yang hanyut melalui langit yang dingin. Sepertinya kecil kemungkinan dia dilahirkan untuk hidup mewah dan mencapai surga setelah kematiannya. Kurang mungkin juga, adalah bahwa dia adalah obyek dari pelayanan Tuhan. ”

198 ““Beberapa orang mengatakan gagasan konspirasi dunia antar generasi untuk menipu massa terdengar tidak masuk akal atau tidak realistis, tetapi orang-orang ini hanya perlu membiasakan diri dengan karya-karya dan tulisan-tulisan Freemason sendiri, misalnya John Robison yang mengekspos ini di tahun 1798-nya. buku, “Bukti dari Konspirasi Terhadap Semua Agama dan Pemerintah Eropa Dilakukan di Pertemuan Rahasia Freemason, Illuminati dan Masyarakat Membaca.” Panglima Tertinggi dari 33 derajat Albert Pike cukup datang dalam beberapa surat tentang Mason tujuan akhir dari dominasi dunia, dan dalam “Protokol Para Tetua Sion yang Dipelajari” Zionis, rencana pasti yang akan menjadi dan telah dilaksanakan sepenuhnya diungkapkan. ”

199“Dari“ Foundations of Many Generations ”oleh E. Eschini,“ Satu hal yang telah dilakukan oleh fabel tentang Bumi yang berputar, telah menunjukkan kekuatan kebohongan yang mengerikan, kebohongan memiliki kekuatan untuk menjadikan seorang pria sebagai budak mental, sehingga dia tidak berani mengembalikan bukti dari indranya sendiri.Untuk menyangkal pergerakan Matahari yang jelas dan jelas yang dilihatnya di hadapannya. Ketika dia merasa dirinya berdiri di Bumi sama sekali tanpa gerak, atas saran orang lain dia siap menerima bahwa dia berputar-putar dengan marah. Ketika ia melihat seekor burung terbang, dan naik ke tanah, ia siap untuk percaya bahwa tanah itu benar-benar bepergian dalam jumlah besar kali lebih cepat daripada burung itu, akhirnya, untuk menegakkan imajinasi orang gila, ia siap untuk menuduh Penciptanya membentuk kebohongan sensif. “

  • TBD(zetetic)

200Dan akhirnya, dari Dr. Rowbotham,” Jadi kita melihat bahwa filsafat Newton ini tidak memiliki konsistensi; rinciannya adalah hasil dari seluruh pelanggaran hukum penalaran yang sah, dan semua premisnya diasumsikan. Kenyataannya, tidak lebih dari asumsi pada asumsi, dan kesimpulan yang diperoleh darinya secara sadar dianggap sebagai sesuatu yang terbukti, dan digunakan sebagai kebenaran untuk memperkuat asumsi pertama dan mendasar. Seperti ‘sulap dan campur aduk’ dari kepiawaian dan kebohongan yang diperpanjang dan diintensifkan seperti dalam astronomi teoritis dihitung untuk membuat penakluk yang tidak berprasangka melawan dengan horor dari sihir yang mengerikan yang telah dipraktekkan kepadanya; untuk bertekad kuat untuk menahan kemajuan lebih lanjut; berusaha keras untuk membuang seluruh bangunan, dan untuk mengubur reruntuhannya penghormatan palsu yang telah dikaitkan dengan perakitnya, dan yang masih melekat pada peminatnya. Untuk pembelajaran, kesabaran, ketekunan dan pengabdian yang pernah mereka contoh, penghargaan dan tepuk tangan tidak perlu ditahan; tetapi alasan keliru mereka, keuntungan yang mereka ambil dari ketidaktahuan umum umat manusia sehubungan dengan mata pelajaran astronomi, dan teori-teori tidak berdasar yang telah mereka tingkatkan dan bela, tidak dapat disesali, dan harus dengan segala cara yang mungkin tercerabut. ”